Questions and Answers



  1.A 72-year-old female presents to the emergency department (ED) with 4 hours of substernal crushing chest pain, that radiates to the back. She reports associated shortness of breath, diaphoresis, and nausea. After getting a thorough history, you realize that her medical history includes hypertension, hyperlipidemia, coronary artery disease with percutaneous coronary intervention, and tobacco use. In the ED an EKG is performed, showing significant ST elevation in EKG leads II, III, and aVF. You act quickly and page the on-call cardiologist and activate the cath lab. While waiting for the cardiology team, you realize that the most likely location of her MI is:


    A.Anterior wall


    B.Lateral wall


    C.Inferior wall


    D.Posterior wall


  Correct Answer C: Inferior wall. Acute inferior wall MI will likely show ST elevation in leads II, III, and aVF. This tells us that the blood vessels geographically located on the inferior part of the myocardium likely are affected. An acute anterior wall MI will show ST elevation in the precordial leads (V1 to V6), whereas a lateral wall MI will have ST elevation in leads I and aVL. A posterior wall MI will be represented by ST depression in the precordial leads (mainly V1 to V4) and may coincide with an inferior wall MI due to the closely shared blood supply in these locations.


  2.A 52-year-old male with a medical history significant for coronary artery disease, hypertension, hyperlipidemia, and diabetes mellitus type II presents to the emergency department with left arm paresthesias, mild back pain, and some dull left-sided chest pain. His 12-lead EKG shows ST elevation in his precordial leads. You quickly realize that reperfusion is crucial for this patient. What are the recommended guidelines for door-to-balloon time from the time the patient enters the emergency room (ER) and receives first medical contact to when he should undergo catheterization?


    A.<30 minutes


    B.<60 minutes


    C.<90 minutes


    D.<120 minutes


  Correct Answer C: <90 minutes. ACCF/AHA guidelines recommend that patients with acute ST elevation myocardial infarction door-to-balloon time be no >90 minutes from the first contact upon arrival to the ER. Patients who are unable to receive percutaneous coronary intervention can receive fibrinolytic therapy if no contraindication. Guidelines recommend that if fibrinolytic therapy is the choice for reperfusion it should be done in no >30 minutes.


  3.A 38-year-old African American female, with no reported significant medical history, presents to the emergency department (ED) with shortness of breath (SOB), which has been worsening over the past 2 days. She has associated pleurisy, cough, and very poor po intake. This morning she believes she saw blood-tinged sputum in a napkin she coughed into. After obtaining a thorough history, you realize that she has recently returned from a business trip to Japan 3 days prior. The only medications she uses are birth control and Advil prn. Her vitals are significant for temp, 98.6°F; HR, 108; BP, 132/80; RR, 24; and O2 sat, 91% on RA. Laboratory studies are significant for white blood cell of 11.2, Cr of 2.3, blood urea nitrogen of 64, and D-dimer of 660. Based on this information, which is the most appropriate diagnostic test that could help to figure out what is going on?


    A.V/Q perfusion scan


    B.Chest CT angiography


    C.Chest x-ray


    D.EKG


  Correct Answer A: V/Q perfusion scan. Based on the provided information, this patient likely has a pulmonary embolism. She is tachycardic and tachypneic, with reported SOB and hemoptysis. She also recently has had prolonged stasis on a trip and is on birth control pills. Her D-dimer is high on initial labs. The best and most sensitive test would be a CTA of the chest to diagnose pulmonary embolism (PE), but with her renal insufficiency, it is contraindicated. A chest x-ray may be helpful to show sequelae of PE, but is nondiagnostic. An x-ray may show atelectasis, cardiomegaly, Hampton’s hump, Westermark’s sign, or possibly effusion. An EKG also would not be diagnostic for PE, and some of the findings that could be present on an EKG with pulmonary emboli can also be present in patients who do not have a PE. If a massive PE is present, it is possible patients may have S1Q3T3 pattern and possibly right ventricular strain or new incomplete right bundle branch block. The most appropriate choice for the diagnosis of PE in this particular patient is a V/Q perfusion scan to check for ventilation mismatch. If the V/Q scan is negative, there is a low probability that this is an acute PE.


  4.A 45-year-old obese male presents to the emergency department (ED) with generalized chest pain, fatigue, and shortness of breath. He has been unable to sleep over the past 2 days since the pain started. He reports the only thing that mildly helps to alleviate the pain is sitting up and leaning forward. When he lies down flat or takes a deep breath, he reports that the pain is intensified. After obtaining a thorough history, you realize that he recently had a Non-ST elevation myocardial infarction 2 weeks prior and had a drug-eluting stent placed to his left anterior descending artery. On exam you note him to have a pericardial friction rub and lungs clear throughout. He is tachycardic on admission, but afebrile. Laboratory studies are significant for white blood cell count of, 13.1; erythrocyte sedimentation rate, 52; troponin; and B-type natriuretic peptide (BNP) normal. Chest x-ray is unremarkable. EKG demonstrates diffuse ST elevation throughout all leads. Based on your findings, the most likely diagnosis is:


    A.ST elevation myocardial infarction


    B.Acute congestive heart failure (CHF)


    C.Myocarditis


    D.Dressler’s syndrome


  Correct Answer D: Dressler’s syndrome. This patient recently had a myocardial infarction with stent placement a few weeks prior to these symptoms. He presents back to the ED with chest discomfort, which is improved with sitting forward. This finding is common in patients with pericarditis, or Dressler’s syndrome (pericarditis following a myocardial infarction [MI]). Other key findings include pericardial friction rub and diffuse ST elevation on EKG. These patients may have nonspecific findings as well, including low-grade fever, leukocytosis, and elevated inflammatory markers. Even though there is ST elevation on the EKG, when the elevations are seen throughout. Diffuse ST elevations on EKG are consistent with pericarditis. No clinical findings support CHF (lungs clear, x-ray normal, BNP normal). This is unlikely myocarditis even though it can present with similar chest pain symptoms such as pericarditis. Myocarditis is also generally associated with troponin elevation. EKG findings may show nonspecific ST-T wave changes, but with myocarditis, there usually is no diffuse ST elevation.


  5.A 74-year-old female is seen in the emergency department for weakness, which has been ongoing for about a week. She denies chest pain, palpitations, or shortness of breath. Medical history includes hypertension, diabetes mellitus type II, and history of rheumatic heart disease. She also reports that her primary care physician told her she has an “abnormal heart exam.” Upon auscultation of her heart, you hear a high-pitched systolic murmur that is best heard at the apex of the heart. Based on these findings, which is the most likely type of murmur present?


    A.Mitral stenosis


    B.Mitral regurgitation


    C.Aortic stenosis


    D.Aortic insufficiency


  Correct Answer B: Mitral regurgitation. Rheumatic heart disease can affect all valves of the heart, but most commonly affects the mitral valve. Mitral regurgitation is a high-pitched, blowing, holo-systolic murmur heard best at the apex. Mitral stenosis is a low-pitched, mid-diastolic rumble murmur heard best at the apex. Aortic stenosis is a midsystolic, crescendo-decrescendo murmur heard best at right upper sternal border. Aortic stenosis also usually is harsh, high-pitched, and radiates to the carotids. Aortic insufficiency is an early diastolic decrescendo murmur heard best at left upper sternal border.


  6.A 74-year-old male with a medical history of colon cancer, status post bowel resection, hypertension, hyperlipidemia, and Parkinson’s disease presents to the emergency department with 2 days of right lower extremity swelling, erythema, and tenderness. On admission his vital signs are stable. A lower extremity ultrasound of the right leg is performed and shows a new deep venous thrombosis. Which of the following is not part of Virchow’s triad?


    A.Venous stasis


    B.Vessel wall injury


    C.Malignancy


    D.Hypercoagulable state


  Correct Answer C: Malignancy. Virchow’s triad describes the three broad categories of factors thought to contribute to thrombosis. These factors are hypercoagulability, hemodynamic changes (stasis), and endothelial injury/dysfunction. Although malignancy is a risk factor for thrombosis, it is not part of this triad.


  7.A 67-year-old female is brought to the emergency department after a car accident. She was the unrestrained driver, and emergency medical services report that she hit a telephone pole and her airbag did not deploy. On admission she reports chest pain and shortness of breath, and she appears to be in significant distress. Vital signs: temp, 98.9°F; HR, 106; BP, 86/54; RR, 26; and O2 sat 92% on RA. You do note that with inspiration there is a 14-mm Hg drop in systolic pressure. Exam is notable for distant heart sounds and faint crackles at the lung bases. Based on this information, what is the gold standard for treatment?


    A.Admit to hospital with urgent cardiology evaluation


    B.Nonsteroidal anti-inflammatory drugs, colchicine, and supportive care with pain medication


    C.Cardiac catheterization


    D.Pericardiocentesis


  Correct Answer D: Pericardiocentesis. This is the mainstay of treatment for cardiac tamponade. This patient will likely need hospital admission and cardiology evaluation, but this is not the mainstay of treatment. Supportive care should be used during the hospital stay for pain control. Cardiac catheterization will have no benefit since there is no vessel occlusion with tamponade, although this is often carried out in the cardiac catheterization laboratory.


  8.A 58-year-old male reports to the emergency department with chest pain and shortness of breath. While obtaining a history, he reports to you that in the past he had open heart surgery complicated by cardiac tamponade. Which is not part of Beck’s triad?


    A.Pulsus paradoxus


    B.Muffled heart sounds


    C.Distended neck veins


    D.Hypotension


  Correct Answer A: Pulsus paradoxus. Beck’s triad is associated with cardiac tamponade, an emergent condition in which fluid accumulates around the heart. It is classified by hypotension, distended neck veins, and muffled heart sounds. Pulsus paradoxus is an abnormally large decrease in systolic blood pressure and pulse wave amplitude during inspiration. Even though this can be found with cardiac tamponade, it is not part of Beck’s triad. Pulsus paradoxus can also be found with pericarditis and obstructive lung disease.


  9.A 58-year-old male with a medical history significant for hypertension and hyperlipidemia presents to the emergency department with acute onset of severe chest and back pain. He reports that the pain is sharp and “takes his breath away.” He reports feeling like he is being ripped open and is unable to tolerate the pain. On exam he appears ill and in severe distress. Vital signs are significant for BP of 84/62 and HR of 112. Based on your initial findings and workup, what would be the most appropriate imaging study to diagnose this potentially life-threatening disorder?


    A.Chest CT with contrast


    B.Transesophageal echocardiogram (TEE)


    C.Coronary angiography


    D.Thoracic MRI


  Correct Answer B: Transesophageal echocardiogram (TEE). Based on initial workup, he likely has an unstable aortic dissection. All of these studies are very sensitive for detection of aortic dissection, but the TEE is the test of choice for any unstable patient and it can be done at bedside.


10.A 64-year-old male, with a known history of hypertension and medical noncompliance, presents to the emergency department with gradual onset of visual changes associated with dizziness and headache. He has not taken any of his home antihypertensives for over a month due to reported financial issues. On admission BP is 188/96 and HR is 85. On fundoscopic exam, which is not a finding associated with hypertensive retinopathy?


    A.Copper/silver wiring


    B.Cotton wool spots


    C.Cherry-red spot in macula


    D.Papilledema


  Correct Answer C: Cherry-red spot in macula. A cherry-red spot found in the macula is not associated with hypertensive retinopathy but rather with central retinal vein occlusion. Copper/silver wiring is found in grade I hypertensive retinopathy, cotton wool spots in grade III, and papilledema in grade IV. Urgent evaluation by an ophthalmologist should occur.


11.A 70-year-old female presents to the emergency department (ED) with back pain for 2 days. She has a medical history significant for hypertension (HTN) and tobacco use. In the ED she is tachycardic, but BP is stable. She appears in acute distress, and on exam, you note a palpable mass in the abdomen. A bedside ultrasound is being ordered, and you are concerned for an aneurysm. Which of the following is not a risk factor for abdominal aortic aneurysm (AAA)?


    A.Diabetes


    B.HTN


    C.Tobacco use


    D.Hyperlipidemia


  Correct Answer A: Diabetes. The risk factors for AAA include smoking (past or current), advancing age, male gender, HTN, hyperlipidemia, atherosclerosis, Caucasian race, family history AAA, or other large artery aneurysm. A decreased risk of AAA is associated with the female gender, diabetes mellitus type II, and non-Caucasian race.


12.A 32-year-old male with a medical history significant for intravenous drug use (IVDU) presents to the emergency department with continued fevers and chills at home for the past 3 days. His last IVDU was 5 days prior, and he uses IV heroin about one to two times per week. Vitals show the following: temp, 101.4°F; HR, 92; and BP, 138/90. On exam you note a murmur heard along the left sternal border. Patient reports he has never been told he has a murmur before. You also note petechiae in his lower extremities as well as raised, red, and tender nodules on his hands. He reports these are also new. Based on your clinical judgment, you are highly concerned for infective endocarditis. He does not have any prosthetic valves, and you order a stat echocardiogram to evaluate for vegetation. You start him on empiric antibiotic therapy with IV vancomycin and gentamicin. What is the most likely organism?


    A.Streptococcus viridans


    B.Enterococcus


    C.Staphylococcus aureus


    D.Staphylococcus epidermidis


  Correct Answer C: Staphylococcus aureus. All of the options listed above are potential organisms found in infective endocarditis. The most common organism associated with IVDU native valve endocarditis is Staphylococcus aureus (about 68%). In IV drug users, the tricuspid valve is most commonly affected. If this were a non-IV drug user, the most common organism would be Streptococcus viridans (about 35%). In non-IV drug user, native valve endocarditis, the most commonly affected valve, is the aortic valve.


13.A 74-year-old female with a history significant for cerebrovascular accident (CVA), hypertension (HTN), and diabetes mellitus (DM) type II presents to the emergency department with acute onset of palpitations. She has experienced similar symptoms a few times over the past few years, but never to this extent. On admission she is tachycardic, with heart rate in the 120s. EKG demonstrates atrial fibrillation with rapid ventricular response. Patient reports that this is a new diagnosis for her. She is curious about the options for treatment. You order a dose of IV diltiazem for rate control, and you discuss anticoagulation with her. Based on the information, what is her CHADS2 score?


    A.3


    B.4


    C.5


    D.6


  Correct Answer B: 4. Based on the provided information, the CHADS2 score is 4. Based on the criteria, 1 point is given for HTN, 1 point for DM, 1 point for age ≥75, 1 point for congestive heart failure, and 2 points for history of transient ischemic attack/CVA/thromboembolism. Guidelines recommend anticoagulation if CHADS2 score is 2 or more unless anticoagulation contraindicated.


14.A 60-year-old male with a history of amyloidosis presents to the emergency department with lower extremity swelling and shortness of breath. His primary care physician told him that he has a “cardiomyopathy,” but he is unclear exactly what this is. On exam you note he has bilateral lower extremity swelling, crackles in his lung bases, and Kussmaul sign noted on exam. What is the most likely type of cardiomyopathy?


    A.Nonischemic hypertrophic cardiomyopathy


    B.Ischemic hypertrophic cardiomyopathy


    C.Dilated cardiomyopathy


    D.Restrictive cardiomyopathy


  Correct Answer D: Restrictive cardiomyopathy. Rigid heart walls that restrict the heart from stretching and filling with blood properly classify this form of cardiomyopathy. Amyloidosis is one of the most common causes of this type of cardiomyopathy. This can easily be confused with constrictive pericarditis, in which Kussmaul sign is also noted (paradoxical rise in jugular venous pressure on inspiration). In amyloidosis clumps of protein build up in body tissues, and over time these proteins replace normal tissue, leading to organ failure. In the heart, this can cause electrical and conduction issues, as well as restrictive cardiomyopathy.


15.A 58-year-old male with a medical history significant for congestive heart failure (CHF), hypertension, and hyperlipidemia presents to the emergency department with shortness of breath (SOB). He reports he cannot lie flat due to difficulty breathing. He also reports waking up frequently at night with SOB and he needs to sleep upright in a recliner. On admission he appears in mild respiratory distress. Chest x-ray shows bilateral pleural effusions with Kerley B lines. Cardiac exam reveals a harsh systolic murmur in the right upper sternal border and he does have lower leg edema. Which of the following medications does not reduce the mortality for CHF?


    A.Lisinopril


    B.Metoprolol


    C.Spironolactone


    D.Lasix


  Correct Answer D: Lasix. Lasix is a loop diuretic which most, if not all, patients with CHF take in acute CHF exacerbations as well as chronically. It is used for symptomatic management but has not proven to decrease mortality. Ace inhibitors, β-blockers, and aldosterone antagonists have shown to decrease overall mortality. Lisinopril will decrease afterload overall, reducing the amount of work the heart has to do. Metoprolol will improve left ventricular function and in the long run also decrease the amount of work the heart has to perform.


16.A 48-year-old female, with no significant medical history, presents to the emergency department complaining of a week of dizziness and fatigue. On admission heart rate is low at 50. Chest x-ray is unremarkable; EKG shows a progressive prolongation of the PR interval on consecutive beats followed by a dropped QRS. After the dropped QRS, the PR interval resets and the cycle repeats. Based on the EKG, which type of heart block is this?


    A.First-degree atrioventricular (AV) block


    B.Second-degree AV block type I


    C.Second-degree AV block type II


    D.Third-degree AV block


  Correct Answer B: Second-degree AV block type I (Mobitz 1, aka Wenckebach). With a first-degree AV block, the PR interval is constantly prolonged (>0.20 seconds) but every QRS is conducted. In second-degree AV block type II (Mobitz 2, aka Hay), the PR interval remains unchanged prior to the P, which suddenly fails to conduct to the ventricles. Third-degree AV block is complete dissociation between P waves and QRS complexes. Both third-degree block and second-degree block type II will likely need pacemaker placement.


17.A 35-year-old male with a past medical history significant for Marfan syndrome presents to the emergency department after a car accident with acute onset of severe chest and back pain. On admission his BP is 156/94 and HR is 76. Chest CTA demonstrates an acute aortic dissection. Which medication should be given first for blood pressure control?


    A.IV labetalol


    B.IV sodium nitroprusside


    C.IV hydralazine


    D.IV nicardipine


  Correct Answer A: IV labetalol. With acute aortic dissection without shock, a β-blocker is used initially to decrease cardiac contractility and to prevent reflex tachycardia. By reducing shear force, it will help to prevent further damage. Reflex tachycardia will increase pressure and worsen the dissection if not prevented/controlled. The goal is to reduce the heart rate to about 60 beats per minute (or lowest tolerated by patient), and the BP should be reduced to mean arterial pressure of 60 or systolic BP of 100 to 120 (or the lowest tolerated by the patient). Sodium nitroprusside is an effective vasodilator with a short half-life and should be used after initial meds to decrease heart rate. Ca channel blockers can be used to decrease systolic BP and contractility in patients that β-blockade is contraindicated (not first-line therapy). Hydralazine is used for BP reduction, but is not a medication used commonly with aortic dissection.


18.A 68-year-old male who presents with chest pain is found to have a ST elevation myocardial infarction (STEMI) in the precordial leads. Unfortunately your hospital does not have reperfusion available. You discuss the different treatment options with the patient and his family and they agree to fibrinolysis. The patient is also in the appropriate time frame to receive the medication. Which of the following is not an absolute contraindication to receiving fibrinolysis?


    A.Intracranial hemorrhage 9 months prior


    B.Ischemic stroke 5 months prior


    C.Active gastrointestinal bleed


    D.Suspected aortic dissection


  Correct Answer B: Ischemic stroke 5 months prior. The absolute contraindications to receiving fibrinolysis therapy for STEMI include any prior intracranial hemorrhage, intracranial neoplasm/arteriovenous malformation/aneurysm, nonhemorrhagic stroke or closed head trauma within 3 months, active internal bleeding or known bleeding diathesis, or suspected aortic dissection. The relative contraindications include history of severe hypertension, ischemic stroke over 3 months, prolonged cardiopulmonary resuscitation, trauma/major surgery within 3 weeks, recent internal bleed/active peptic ulcer disease, noncompressible vascular punctures, pregnancy, or current use of anticoagulants.


19.A 36-year-old male presents with “flu-like symptoms” and is found to be febrile on admission with a temperature of 101.1. EKG shows sinus tachycardia with nonspecific changes; biomarkers are elevated. After further workup, the patient is diagnosed with myocarditis and admitted to the hospital for treatment with supportive care. What is the most common cause of myocarditis?


    A.Idiopathic


    B.Chemotherapy


    C.Connective tissue disease


    D.Infectious


  Correct Answer D: Infectious. Myocarditis is an inflammation of the myocardium and commonly is associated with pericarditis. Even though all the choices are causes for myocarditis, the most common cause is viral infection (particularly coxsackie B virus). The treatment for myocarditis typically is supportive care.


20.A 62-year-old female presents to the emergency department for worsening leg swelling. On exam you note she has pitting edema in her lower legs. She also has mild right upper quadrant pain and jugular venous distention (JVD). EKG is unremarkable; chest x-ray shows cardiomegaly; B-type natriuretic peptide is elevated. What is the most common cause of right-sided heart failure?


    A.Left-sided heart failure


    B.Left-to-right shunt


    C.Pulmonary embolism


    D.Coronary artery disease


  Correct Answer A: Left-sided heart failure. When the left ventricle fails, increased fluid pressure is transferred back through the lungs, ultimately damaging the heart’s right side. With a left-to-right shunt, the right ventricle (RV) eventually becomes volume overloaded. With pulmonary emboli and chronic lung disease, high blood pressure in the pulmonary arteries will cause RV failure, and with coronary artery disease, if there is a right-sided infarct, the right ventricle will fail and cause blood to back up into the systemic system.


21.A 58-year-old male with a history of coronary artery disease, hypertension, hyperlipidemia, diabetes mellitus type II, and tobacco use presents to the emergency department with left arm paresthesias, numbness, and dull left-sided chest pain. He also reports dyspnea on exertion, nausea, and diaphoresis. On admission his vitals are stable. EKG shows new ST elevation in V1 to V6. Based on the type of myocardial infarction (MI), which coronary artery likely is blocked?


    A.Right coronary artery


    B.Left anterior descending artery


    C.Right circumflex artery


    D.Left circumflex artery


  Correct Answer B: Left anterior descending artery. With ST elevation in V1 to V6, this would represent an anterior wall MI. Based on the anatomical location of the heart, this would correlate with the left anterior descending artery (LAD). The LAD is a branch off the left main coronary artery. The LAD supplies the anterior and inferior portions of the heart and thus would be the affected vessel in an anterior wall MI. The left circumflex artery (LCX) is also a branch of the left main coronary artery. The LCX supplies blood to the left atrium and lateral/posterior portions of the left ventricle. It will be affected most commonly in a lateral wall MI. The right coronary artery (RCA) supplies blood to the right atrium, right ventricle, and the bottom portion of both ventricles and the back of the septum. The RCA most commonly is affected in inferior wall MI or right ventricular MI.


22.A 46-year-old male presents to the emergency department (ED) with palpitations and fatigue. In the ED his HR is elevated to the 160s. 12-lead EKG demonstrates narrow-complex supraventricular tachycardia. Since vital signs are stable, you attempt vagal maneuvers that are unsuccessful. Which is the first medication that should be administered?


    A.Adenosine 12 mg


    B.Adenosine 6 mg


    C.Atropine 0.5 mg


    D.Epinephrine 1 mg


  Correct Answer B: Adenosine 6 mg. This patient has a stable tachyarrhythmia on admission. He is found to have a narrow-complex tachyarrhythmia, and since vagal maneuvers are unsuccessful, medical therapy should be used to break the arrhythmia. If this patient were unstable then cardioversion should be performed immediately. The first dose of adenosine is 6 mg, and if this is unsuccessful, the second dose is 12 mg. Atropine is used for bradyarrhythmias, and epinephrine is used in cardiac arrest.


23.A 58-year-old female is brought to the emergency department by her husband complaining of palpitations. On admission to the ED her HR is 146 and BP is 80/54. She starts to report chest discomfort on admission, and she appears fatigued and clammy. You immediately hook her up to a cardiac monitor, administer O2, and obtain IV access. 12-lead EKG shows atrial fibrillation with rapid ventricular response. What is the initial treatment of choice?


    A.Synchronized cardioversion


    B.IV diltiazem


    C.IV labetalol


    D.Vagal maneuvers


  Correct Answer A: Synchronized cardioversion. This is indicated for treatment of unstable supraventricular tachycardia, unstable atrial fibrillation, and unstable atrial flutter. Since this patient is hypotensive with chest discomfort, she is considered unstable and needs cardioversion. Both IV diltiazem and labetalol can be used for rate control in atrial fibrillation once stable. Vagal maneuvers should be attempted in stable wide QRS tachyarrhythmias only.


24.A 28-year-old male presents to the emergency department with dyspnea. He reports he recently had an echocardiogram as an outpatient and was diagnosed with hypertrophic cardiomyopathy. What type of murmur would you expect to hear on exam?


    A.Diastolic murmur that increases in intensity with Valsalva and standing


    B.Systolic murmur that increases in intensity with Valsalva and standing


    C.Diastolic murmur that decreases in intensity with Valsalva and standing


    D.Systolic murmur that decreases in intensity with Valsalva and standing


  Correct Answer B: Systolic murmur that increases in intensity with Valsalva and standing. Hypertrophic cardiomyopathy (HOCM) is a disease resulting in abnormal myocardium thickening commonly in the interventricular septum. This is a concerning cardiomyopathy since it can result in sudden cardiac death. HOCM is also an autosomal dominant genetic disorder. On exam the murmur is a high-pitched, crescendo-decrescendo, systolic murmur heard best at left lower sternal border. This murmur does not radiate to the carotids (unlikely aortic stenosis). The murmur with HOCM becomes more intense with Valsalva and standing. These maneuvers decrease left ventricular filling, resulting in worsened left ventricular outflow tract obstruction since the septum is thickened making the murmur louder. With aortic stenosis, the murmur becomes softer with these positions since less blood is being ejected through the aortic valve.


25.A 57-year-old male presents to the emergency department after being sent from his primary care physician for uncontrolled hypertension (HTN). He reports a history of HTN but he has been noncompliant with his home antihypertensive medications. His BP is elevated on initial vitals. Which of the following represents hypertensive urgency?


    A.BP of 210/112 with papilledema


    B.BP of 168/122 with headache


    C.BP of 190/86 with acute renal failure


    D.Pregnant female with BP of 150/86


  Correct Answer B: BP of 168/122 with headache. Hypertensive crises can present as hypertensive urgency or emergency. Hypertensive urgency is a condition defined by elevated BP (≥180 systolic and/or ≥110 diastolic), without evidence of end-organ damage. The patient may experience minor symptoms, including headache, shortness of breath, or anxiety. Hypertensive emergency is a condition marked by elevated BP with evidence of end-organ damage. This includes stroke, change in mental status, myocardial infarction, acute renal failure, aortic dissection, angina, pulmonary edema, or eclampsia. For hypertensive urgency, treatment should be to decrease BP over hours with oral meds if possible (target to normalize BP in 1 to 2 days). When there is concern for hypertensive emergency, treatment should aim to decrease mean arterial pressure by about 25% in minutes to 2 hours with IV antihypertensives.


26.A 10-year-old male presents to the emergency department with his mother with lethargy, fever, nausea, and proptosis. His mother reports that for the past 5 days he has been complaining of nasal drainage and congestion, but no reports of cough for sore throat. On exam the patient is lethargic and febrile to 101°F. Head, eyes, ears, nose, and throat exam is significant for no sinus tenderness, nonerythematous tonsils without exudate. What is the likely cause?


    A.Acute sinusitis


    B.Cavernous sinus thrombosis (CVT)


    C.Meningitis


    D.Urinary tract infection (UTI)


  Correct Answer B: Cavernous sinus thrombosis (CVT). CVT usually accompanies an untreated odontogenic or sinus infection sources. Acute sinusitis may have been the predisposing infection; however, it does not cause cranial nerve deficits. With fever and neurologic deficits on exam, meningitis is a likely answer; however, with the known history, it is lower on the differential. UTI, though part of the workup in a febrile pediatric patient, does not present with the following symptoms, history, or exam findings.


27.A 25-year-old female presents to the ED complaining of a sore throat, difficulty swallowing, and fever. She was recently seen in an urgent care 2 days ago and diagnosed with Streptococcus pharyngitis and started on a course of penicillin, but reports no improvement in her symptoms. She reports taking ibuprofen 600 mg 1 hour prior to presentation. On exam her Vital signs: temp, 99.0°F; HR, 90; BP; 110/50; RR, 20; and O2 sat 99% on RA. She looks ill, but not toxic appearing. Throat exam reveals normal-appearing tonsils without erythema or exudate. What is the next likely course of action to be taken?


    A.Continue course of antibiotics and follow-up with her primary care physician in 3 days


    B.Send a throat culture


    C.Order a CT scan of the neck


    D.Admit the patient for IV antibiotics


  Correct Answer C: Order a CT scan of the neck. This is done to assess for a retropharyngeal abscess. The index of suspicion should be high as a patient with a retropharyngeal abscess may have a normal physical exam. The patient usually has improvement 24 hours after the start of antibiotic therapy. Even so, with worsening symptoms, the differential should be broadened for other causes. A throat culture may have been appropriate during the initial urgent care visit, but would not be the next step on this ED visit, as management would not be altered. Admission with IV antibiotics and ENT consult for I+D in the operating room is the ultimate disposition of the patient, and is not the next likely step in this patient’s workup.


28.What is the antibiotic of choice for the treatment of Streptococcus pharyngitis in an adult patient who is allergic to penicillin and macrolide antibiotics?


    A.Clindamycin


    B.Ciprofloxacin


    C.Cefadroxil


    D.Erythromycin


  Correct Answer C: Cefadroxil. For a patient who is penicillin allergic, the current recommendations would be to start a macrolide antibiotic, such as erythromycin or azithromycin. In a patient who is both penicillin and macrolide allergic, the next antibiotic of choice is a first-generation cephalosporin.


29.A 65-year-old male presents to the emergency department complaining of sudden onset of right eye pain. He reports decreased vision, photophobia, and pain. The patient has had bilateral cataract surgeries and has since had fixed pupils on exam. When concerned for an acute ophthalmologic emergency, what is the next exam test that should be performed?


    A.Visual acuity


    B.Slit lamp exam


    C.Measurement of intraocular pressure


    D.Fluorescence staining


  Correct Answer C: Measurement of intraocular pressure. The most concerning diagnosis for this patient’s symptoms would be acute narrow-angle glaucoma, and as such, measurement of intraocular pressure would be the next test that should be performed. A slit lamp exam should be done to assess for cells in the anterior chamber in uveitis, which may lead to glaucoma, but in acute narrow-angle glaucoma, delay of treatment may result in complete vision loss. The patient did not have any history of trauma to suspect a corneal abrasion. A visual acuity should be done on all patients presenting with an eye complaint.


30.A 28-year-old female presents after being struck in the left eye with a baseball. Exam is notable for normal visual acuity, periorbital ecchymosis, and blood in the anterior chamber. You diagnose her with a hyphema. What is the most appropriate treatment?


    A.Warm compresses to the affected eye


    B.Nonsteroidal anti-inflammatory drugs (NSAIDs) to reduce inflammation and to treat pain


    C.Rest with the head of the bed elevated


    D.No further treatment recommended


  Correct Answer C: Rest with head of bed elevated. Most cases of hyphemas resolve on their own, but current recommendations are to rest with the head of the bed elevated. Warm compresses are not contraindicated, but are not the most appropriate treatment. NSAIDs should be avoided to prevent prolonged bleeding.


31.An 18-year-old male presents to the emergency department complaining of pain and redness in the left eye, which started when he woke up. He has no history of any sexually transmitted diseases. The patient admits to regularly wearing his soft contact lens while sleeping. Slit lamp examination reveals significant conjunctival injection and discharge. Fluorescein staining shows a large oval defect in the central cornea with an underlying white haze. What is the most likely diagnosis?


    A.Herpes simplex keratitis


    B.Corneal abrasion


    C.Corneal ulcer


    D.Conjunctivitis


  Correct Answer C: Corneal ulcer. The findings on the slit lamp exam, combined with his history of contact lens use, prove this diagnosis, which is an ophthalmic emergency. Herpes simplex keratitis will show dendritic lesions on slit lamp. Corneal abrasion is a very superficial injury seen on slip lamp. Conjunctivitis can have a red, irritated eye, but it is usually associated with drainage and without defects seen on slit lamp exam.


32.A patient presents complaining of progressive left ear hearing loss. On exam there is impacted cerumen in the affected ear. What other physical exam findings would you have in determining conductive hearing loss?


    A.On the Weber test, sound lateralizes to the unaffected ear and sound is heard longer through the bone than through air.


    B.On the Rinne test, sound is heard longer on the bone than through air to the affected ear and sound does not lateralize.


    C.On the Rinne test, sound is heard longer through the bone than through air to the affected ear and sound lateralizes to the affected ear.


    D.On the Weber test, sound lateralizes to the affected ear and sound is heard longer through air than through bone.


  Correct Answer D: On the Weber test, sound lateralizes to the affected ear and sound is heard longer through air than through bone. In conductive hearing loss, the sound lateralizes to the affected ear on the Weber test and sound is heard longer through the bone than through air on the Rinne test. In sensorineural hearing loss, the sound lateralizes to the unaffected ear on the Weber test and sound is heard longer through air than through bone.


33.A 68-year-old male presents complaining of right eyelid swelling and redness. He denies any fevers, changes in vision, and only minimal irritation. On exam there is erythema and swelling to his upper right eyelid with normal conjunctivae. What is the most appropriate next step?


    A.Discharge the patient home advising warm compresses and frequent washing


    B.Start the patient on oral antibiotics


    C.Order a CT scan


    D.Consult an ophthalmologist


  Correct Answer A: Discharge the patient home advising warm compresses and frequent washing. The patient likely has blepharitis, which can be managed with proper eyelid hygiene. Oral antibiotics can be used, but generally blepharitis resolves without antibiotic use. If an antibiotic were to be used, a topical antibiotic would be the initial choice. Oral antibiotics are indicated with early periorbital cellulitis; however, there were no physical exam findings consistent with that diagnosis. CT imaging would be necessary if concerned for orbital cellulitis as well as consulting ophthalmology, but in a patient presenting with isolated eyelid erythema and edema, without concerning exam findings, blepharitis is the likely diagnosis.


34.In a person diagnosed with whooping cough, at what stage are they infectious?


    A.Incubation stage


    B.Catarrhal stage


    C.Paroxysmal stage


    D.Convalescent stage


  Correct Answer A: Incubation stage. A person can spread pertussis within the first 3 weeks of the start of cough, which begins in the catarrhal stage. During the incubation stage, patients are typically asymptomatic. Patients are no longer considered infectious after 5 days of antibiotics, or after the 3rd week from symptom onset.


35.A 50-year-old female with a history of atrial fibrillation, who is not on anticoagulation, presents with a complaint of sudden painless left eye vision loss. What physical exam finding are you most expecting to see on the fundoscopic exam?


    A.Cotton wool spots


    B.Irregular contours of the retina


    C.Cherry-red spot


    D.Cloudy lens


  Correct Answer C. Cherry-red spots. A person with atrial fibrillation who is not on anticoagulation is at risk for a thrombotic event. A blanched retina and cherry-red spot are the classic physical exam findings. Cotton wool spots may be present, but are not associated with sudden onset of vision loss and are indicative of other disease states, such as hypertension or diabetes. Retinal detachments can also cause sudden painless vision loss; however, as discussed above, this person is at risk for a thrombotic event. A cloudy lens is seen in cataracts and is the most common cause for progressive painless vision loss.


36.A 55-year-old male presents for a vesicular lesion on the tip of his nose. This lesion was preceded by generalized fatigue and subjective fevers. What is an important test to do in the evaluation of this patient?


    A.Skin biopsy


    B.Otoscopic exam


    C.Slit lamp exam


    D.Tonometry


  Correct Answer C: Slit lamp exam. In a patient with a vesicular lesion on the tip of his nose, the differential diagnosis includes herpes zoster. The tip of the nose is located in the V1 branch of cranial nerve V (trigeminal nerve), and an evaluation for herpes zoster ophthalmicus must be done. Timely diagnosis and referral to an ophthalmologist for persistent symptoms are necessary to limit visual morbidities. A skin biopsy can be done to help aid in diagnosis; however, most cases are clinically diagnosed based on history and physical exam. Additionally, if a patient is in need of a skin biopsy, proper dermatology referral should be performed. A slit lamp exam without staining will not reveal the dendritic lesions seen in herpes zoster ophthalmicus.


37.What criteria give you a negative value when evaluating for Streptococcus pharyngitis using the Centor score?


    A.Absence of cough


    B.Age 14 to 44


    C.Age >45


    D.Age <2


  Correct Answer C: Age >45. Age >45 gives you a −1 value. Absence of cough gives you +1, age >15 but <44 gives you 0, and the rule cannot be applied to patients <2 years of age.


38.A 25-year-old female presents complaining of pain, swelling, and drainage from her right ear for the past 5 days. She reports decreased hearing, but denies fevers. On exam she has tenderness with palpation of the tragus, drainage from the ear, an erythematous and edematous canal, clear tympanic membranes, no mastoid tenderness, and no trismus. What is the next likely course of treatment?


    A.CT scan


    B.ENT consultation


    C.Oral antibiotics


    D.Topical antibiotics


  Correct Answer D: Topical antibiotics. The patient has otitis externa and should be started on topical otic antibiotics. TMs are clear and, therefore, do not suggest otitis media. A CT scan should be done if suspecting mastoiditis, but in a well-appearing patient without mastoid tenderness, it is less likely. An ENT consultation should be done for patients who have refractory otitis externa despite a complete course of topical antibiotics.


39.A 2-year-old girl presents with her father with 1 week of purulent drainage/discharge from the right nostril. No reports of fevers or coughs. The father reports a foul odor from the nose. What is the likely diagnosis?


    A.Acute sinusitis


    B.Upper respiratory infection


    C.Nasal polyp


    D.Retained foreign body


  Correct Answer D: Retained foreign body. Without infectious symptoms or associated symptoms consistent with upper respiratory infection, the likely cause is a retained foreign body.


40.The steeple sign is associated with what disease state?


    A.Croup


    B.Epiglottis


    C.Retropharyngeal abscess


    D.Pertussis


  Correct Answer A: Croup. The thumb print sign is associated with epiglottitis. A patient with a retropharyngeal abscess narrowing of the retropharyngeal space may be seen on plain films of the neck, but better seen on a CT scan. Pertussis does not have an associated radiographic sign.


41.A 37-year-old male presents to the emergency department after having been bitten by a bat that was in his basement. He appears well; vital signs include a temp, 98.4°F, HR, 85; and BP, 136/84. He has no medical history and no history of rabies exposure or rabies vaccination. His wound is without significant erythema or tenderness. What would be the most important intervention before discharging him from the emergency department?


    A.IV antibiotics to cover for animal bites


    B.One dose of intramuscular human rabies immune globulin (HRIG) and initiation of rabies vaccine with follow-up instruction to complete the series


    C.Initiation of rabies vaccine with follow-up instruction to complete the series


    D.Irrigation of the wound and debridement if necessary


  Correct Answer B: One dose of intramuscular human rabies immune globulin (HRIG) and initiation of rabies vaccine with follow-up instruction to complete the series. Bats are high-risk animals carrying rabies; patients who are exposed need to receive the vaccine series. The patient should have HRIG and the vaccine on day one. The wound is not being described as infected; therefore, the patient does not need IV antibiotics or debridement.


42.A 19-year-old female arrives to the emergency department brought in by her boyfriend, complaining of diffuse lower abdominal pain and purulent/bloody vaginal discharge. Vital signs: temp, 102.7°F; HR, 107; and BP, 105/75. She has a history of multiple sexual partners without the use of barrier protection/birth control. Initial testing shows a negative urinalysis and the patient is not pregnant. She is diagnosed presumptively with pelvic inflammatory disease (PID), and cultures are sent for analysis. What would be the most appropriate treatment?


    A.Ceftriaxone 250 mg intramuscular (IM) × 1 dose plus doxycycline 100 mg po bid × 14 days


    B.Azithromycin 1 g po × 1 dose plus ceftriaxone IM × 1 dose


    C.Metronidazole 500 mg po tid × 10 days


    D.Do not treat, wait for culture results


  Correct Answer A: Ceftriaxone 250 mg IM × 1 dose plus doxycycline 100 mg po bid × 14 days. This is the standard treatment for PID. Azithromycin plus ceftriaxone is the treatment when you suspect chlamydia or gonorrhea cervicitis. Metronidazole 500 is the treatment for vaginal trichomoniasis. It is not recommended to wait for cultures in any suspected sexually transmitted disease case.


43.What is the most common sexually transmitted disease in the United States?


    A.HPV (human papillomavirus)


    B.Gonorrhea


    C.Chlamydia


    D.Syphilis


  Correct Answer A: HPV (human papillomavirus). Previously chlamydia had been the most common STD in the United States; however, the incidence for HPV has increased, making it the most common existing STD in the United States. Gonorrhea and Syphilis are less common.


44.A 45-year-old presents to the ED with complaints of fevers for 2 days and development of a rash over the last 24 hours. The rash is vesicular with a “dew drop on a rose petal” appearance noted on the bilateral upper and lower extremities as well as the trunk. The rash is pruritic and spares the palms and soles. What is your treatment plan?


    A.Admit for broad-spectrum IV antibiotics


    B.Doxycycline 100 mg po bid × 3 weeks


    C.Admit for supportive treatment, including Tylenol and IV Fluids


    D.Acyclovir 800 mg po qid × 5 days


  Correct Answer D: Acyclovir 800 mg po qid × 5 days. This patient has a rash consistent with chickenpox. Acyclovir is the treatment in adult patients. In children, supportive treatment is the mainstay of therapy; however, it is recommended to treat adults with an antiviral medication. Antibiotics are not warranted, as this is not a bacterial rash. Doxycycline is incorrect because this is not a tick-borne illness.


45.A 19-year-old male presents to the emergency department with complaints of sore throat and generalized malaise. On exam the patient has exudates on tonsils and posterior cervical lymphadenopathy. What laboratory finding will help confirm the diagnosis?


    A.Elevated liver function tests (LFTs)


    B.Positive cold agglutinin test


    C.Positive heterophile antibody


    D.Elevated C-reactive protein


  Correct Answer C: Positive heterophile antibody. This patient most likely has infectious mononucleosis. A positive heterophile antibody is the most specific test confirming the diagnosis. Although a patient may have elevated LFTs and an elevated C-reactive protein, these tests are not specific to mono. A positive cold agglutinin test does not assist in the diagnosis of infectious mononucleosis.


46.A 24-year-old male presents to the emergency room with diarrhea for the past 2 weeks. His primary care physician evaluated him 2 days previously and no antibiotics were started. Vital signs: temp, 100.5°F; HR 105; BP, 98/65; and patient appears profoundly dehydrated. He describes profuse watery diarrhea with generalized malaise and intermittent fevers. He has no medical history, but admits to recently returning from Africa, where he was doing missionary work. Laboratory studies are notable for the following: white blood cell, 22; Na, 132; K, 2.9; and creatinine, 1.80. Stool cultures sent by his PCP earlier this week have returned confirming Giardia. What is the next most appropriate treatment plan?


    A.Admit for symptomatic treatment intravenous fluids (IVF) and antidiarrhea medication


    B.Discharge home with continued supportive therapy


    C.Admit for IVF and metronidazole IV or po 500 mg bid


    D.Discharge home with a script for metronidazole 500 mg po bid × 5 days


  Correct Answer C: Admit for IVF and metronidazole IV or po 500 mg bid. The patient requires antibiotic therapy for symptomatic diarrhea and admission for IVF for associated acute renal failure.


47.What opportunistic infection is an HIV patient most at risk to develop when CD4 counts drop to 190 µL?


    A.Pneumocystis jiroveci (carinii) pneumonia


    B.Toxoplasmosis


    C.Histoplasmosis


    D.Mycobacterium avium complex (MAC)


  Correct Answer A: Pneumocystis jirovecii (carinii) pneumonia. Patients with CD4 counts <200 µL are at risk for P. jiroveci (carinii) pneumonia. Patients with CD4 counts <100 µL become higher risk for developing toxoplasmosis and histoplasmosis. When CD4 counts drop below 50 µL, the patient is at risk for MAC.


48.A 23-year-old active male presents to the ED with complaints of a rash. He began feeling joint pain and having fevers 2 days ago. His rash is diffuse, maculopapular, and is noted on his palms and soles. Upon further questioning, he reveals that he is an avid hiker and spent time camping in the mountains last weekend. What is the treatment of choice?


    A.Supportive care and NSAIDs


    B.Doxycycline 100 mg po bid


    C.Acyclovir 800 mg po qid


    D.Topical steroids


  Correct Answer B: Doxycycline 100 mg po bid. This is classic Rocky Mountain spotted fever. Treatment is doxycycline. Steroids and Acyclovir are not indicated in this diagnosis.


49.Which of the following does not put a patient at risk for transmission of hepatitis B?


    A.Contact with blood or open sores of an infected person


    B.Sex with an infected partner


    C.Breast-feeding from an infected person


    D.Birth to an infected mother


  Correct Answer C: Breast-feeding from an infected person. The Centers for Disease Control and Prevention guidelines state that hepatitis B can be transmitted through blood, bodily fluids, and mucous membranes, though not through breast milk.


50.A 21-year-old male college student presents to the emergency department with headache, photophobia, and generalized malaise. His intake vital signs include the following: temp, 102°F; HR, 110; BP, 100/62; and RR, 18. He appears toxic and uncomfortable. Upon examination he has nuchal rigidity. You suspect meningitis and perform a lumbar puncture. Cerebrospinal fluid cell count reveals white blood cell 1,300, protein 21, and glucose 30. What is the most likely bacterial source?


    A.Group B Streptococcus


    B.Escherichia coli


    C.Listeria monocytogenes


    D.Streptococcus pneumoniae


  Correct Answer D: Streptococcus pneumoniae. This patient has meningitis. S. pneumoniae and Neisseria meningitidis are the most common types of bacteria seen in young adults as well as older adults. Group B Streptococcus, E. coli, and L. Monocytogenes are more common in newborns.


51.Which of the following will help support the diagnosis of viral meningitis when interpreting cerebrospinal fluid results?


    A.Predominance of lymphocytes


    B.Markedly decreased glucose


    C.Elevated opening pressure


    D.Elevated white blood cell


  Correct Answer A: Predominance of lymphocytes. On a cell count/differential, there is typically a predominance of lymphocytes on patients with viral meningitis. With viral meningitis, you will typically see a normal glucose level with normal opening pressure.


52.A 67-year-old, with a medical history of benign prostatic hyperplasia and hypertension, presents to the emergency room with complaints of shortness of breath. His initial vital signs include the following: temp, 100.5°F; HR, 105; BP, 92/45; and RR, 25. Basic labs show white blood cell 12, creatinine 0.9, blood urea nitrogen 8, and C-reactive protein 12. His chest x-ray reveals left lower lobe pneumonia. The patient appears more comfortable after an albuterol nebulizer treatment and 1 L of intravenous fluids. What is the most appropriate intervention for this patient?


    A.Discharge the patient home with an antibiotic to treat of community-acquired pneumonia


    B.Discharge the patient home with a nebulizer machine, albuterol, and an antibiotic to treat of community-acquired pneumonia


    C.Admit the patient to the general medical floors, and initiate treatment for community-acquired pneumonia


    D.Admit the patient to the intensive care unit, and initiate broad-spectrum antibiotics to treat for hospital-acquired pneumonia


  Correct Answer C : Admit the patient to the general medical floors, and initiate treatment for community-acquired pneumonia. The CURB-65 is often used to determine the risk of mortality in patients with pneumonia for clinical practice. This particular patient scores a 3 (1 point for increased respiratory rate, 1 point for low blood pressure, and 1 point for age >65). It is recommended to admit the patient with a score of 2 or more. However, given the patient’s medical history, he does not appear to be at risk for hospital-acquired pneumonia.


53.A 32-year-old female with a medical history of asthma presents to the emergency department with fevers up to 102, chills, and a productive cough. Her chest x-ray confirms pneumonia. What is the most likely pathogen?


    A.Streptococcus pneumoniae


    B.Haemophilus influenzae


    C.Chlamydophila pneumoniae


    D.Staphylococcus aureus


  Correct Answer A: Streptococcus pneumoniae. This is the most common cause of community-acquired pneumonia. H. influenzae, C. pneumoniae, and Staphylococcus aureus are also common causes of pneumonia; however, they are not the most common types seen in the community.


54.Which of the following is true regarding influenza treatment?


    A.Children under the age of 2 years, but older than 14 days are considered high risk and should receive Tamiflu (oseltamivir)


    B.Initiating Tamiflu (oseltamivir) has no benefit for a patient 48 hours after the onset of symptoms


    C.Tamiflu (oseltamivir) should not be initiated in pregnancy


    D.Tamiflu (oseltamivir) should not be initiated in immunosuppressed patients


  Correct Answer A: Children under the age of 2 years, but older than 14 days are considered high risk and should receive Tamiflu (oseltamivir). Children under 2 years old are considered high risk for developing complications of pneumonia. The Food and Drug Administration approved the drug for children older than 14 days in 2012. Tamiflu is safe in pregnancy and should be initiated in high-risk populations, including immunosuppressed patients. Although initiating the drug <48 hours of symptom onset shows the greatest benefit, however, there is still a benefit to initiating treatment after 48 hours of symptom onset.


55.A 24-year-old female with no medical history presents to the emergency department with an acute onset of right-sided facial weakness. She is unable to completely close her right eye and is worried that she may have suffered a stroke. She also admits to having a circular rash of the back of her neck, which has now resolved. She has an otherwise normal neurologic examination with 5/5 strength of all the extremities. Upon further questioning, she tells you that she has recently done landscaping work in her yard. What is the next most appropriate intervention?


    A.Admit to the hospital for stroke workup, including an MRI/MRA of her head and neck


    B.Obtain blood cultures, and admit to the hospital for treatment with IV antibiotics for septic arthritis


    C.Obtain Lyme serology, and initiate doxycycline to treat Lyme disease


    D.Obtain basic labs, and initiate acyclovir to treat Bell palsy


  Correct Answer C: Obtain Lyme serology, and initiate doxycycline to treat Lyme disease. Based on this patient’s history and physical exam, she most likely has Lyme disease. Although facial palsies may resolve without treatment, oral antibiotic therapy may prevent further sequelae. This patient’s signs and symptoms are not consistent with Lyme meningitis or encephalitis, which would require 28 days of IV antibiotics (generally a cephalosporin). Although a workup for this patient may include imaging or blood cultures, it is not the most important next step for this patient’s work up. The most likely source for the patient’s Bell’s palsy is Lyme and not a viral source; so acyclovir is not indicated.


56.A 22-year-old female, without any medical history, presents to the emergency department complaining of right-sided facial weakness that was first noticed about 8 hours ago. Her only complaint is that she is having difficulty closing her right eye, but otherwise feels well. She has no other complaints. The exam is only significant for obvious facial weakness, and patient is unable to wrinkle her forehead. What is your next step?


    A.MRI/MRA head and neck


    B.Start prednisone and discharge home


    C.Stat tPA


    D.Carotid ultrasound


  Correct Answer B: Start prednisone and discharge home. This patient has a history and exam consistent with Bell’s palsy. Bell’s palsy is a clinical diagnosis MRI/MRA and carotid ultrasound are not as helpful; however, in older patients with more risk factors for a stroke, imaging could be considered to rule out cerebrovascular accident (CVA). This is not an acute stroke given her lack of other findings, hence tPA, which would be used for acute stroke, is not indicated.


57.What physical exam finding would you expect to find in a patient with Bell’s palsy?


    A.Positive pronator drift


    B.Inability to elevate right side of forehead


    C.Carotid bruit


    D.Anisocoria


  Correct Answer B: Inability to elevate right side of forehead. Bell’s palsy is a motor seventh cranial nerve palsy; so patients are unable to elevate or raise their forehead. You would otherwise have a normal neurologic exam. Carotid bruit is not something you would typically find in Bell’s palsy.


58.A 74-year-old male with a history of hypertension, atrial fibrillation, and hyperlipidemia is complaining of acute onset of severe headache. Patient states that this headache is much worse than his typical headaches. Patient is awake, alert, but appears to be uncomfortable and in mild distress. Vital signs: temp, 97.8°F; HR, 96; BP, 192/104; RR, 22; and O2 sat 96% on RA. A head CT confirms a subarachnoid hemorrhage (SAH). Neurosurgery has been consulted. Which of the following is the most important next step?


    A.Start nicardipine drip


    B.Start Keppra


    C.Intubate


    D.Obtain an MRI/MRA


  Correct Answer A: Start nicardipine drip. Blood pressure control is the most important next step in a patient with a SAH. MRI/MRA, although may be warranted, will most likely take several hours and is not the most important next step. Patient is awake, alert, and maintaining his airway; so there is no indication to intubate at this time. Starting Keppra to prevent seizures is indicated, but not as important as blood pressure control.


59.An 89-year-old male, with a history of prostate cancer, is transferred to the ED from his nursing home with mental status change. The patient has become more confused, aggressive, and combative ever since being discharged from the hospital where he was admitted for 2 days for urinary retention. While in the hospital, he had a Foley placed and was started on Flomax. Today in the ED, vital signs are significant for temp, 102.1°F; HR, 116; BP, 95/54; RR, 24; and O2 sat 97% on RA. What is the most likely cause for this patient’s mental status change?


    A.Medication effect


    B.Urinary tract infection (UTI)


    C.Dehydration


    D.Metastatic disease to the brain


  Correct Answer B: Urinary tract infection (UTI). The presence of fever, tachycardia, and hypotension likely indicates infection, and the recent Foley placement makes him high risk for UTI. The other options are possible causes of mental status change; however, his history and vital signs make UTI much more likely.


60.A 16-year-old female is brought in by emergency medical services after having her first seizure. She was witnessed to have a 2-minute tonic-clonic seizure with urinary incontinence. There was no head trauma. In the ED patient is postictal, but no focal neurologic findings. Vital signs: temp, 99°F; HR, 67; BP, 110/54; RR, 16; O2 sat 99% on RA. All of the following are indicated in the ED except:


    A.EEG


    B.Chest x-ray


    C.Head CT scan


    D.Complete blood count, Chem 7, toxicology screen, urinalysis, hCG


  Correct Answer A: EEG. Chest x-ray, lab work, and head CT are all indicated in the ED to rule out infection, toxic metabolic disorder, intracranial bleed, edema, or mass etiologies for first-time seizures. EEGs are not routinely done in the ED and obtained by the outpatient neurologist during follow-up.


61.All are findings you could find in a patient with a cerebellar infarct except:


    A.Gait disturbance


    B.Dysmetria


    C.Nystagmus


    D.Visual field defect


    E.Vomiting


  Correct Answer D: Visual field defect. Visual field defects are noted in middle cerebral artery strokes. All other options are all typical findings in a patient with a cerebellar infarct.


62.A 24-year-old female comes to the emergency department (ED) complaining of severe left-sided headache, left eye pain, and left facial numbness. The headache is throbbing and associated with nausea and vomiting. She also complains of “seeing spots.” She has had similar headaches in the past, but she has never seen a specialist or been given a diagnosis for these symptoms. Her only medication is levothyroxine. Her vital signs are as follows: temp, 99.2°F; HR, 100; BP, 120/88; RR, 24; and O2 sat 100% on RA. Her neuro exam is nonfocal. What is her most likely diagnosis?


    A.Bell’s palsy


    B.Transient ischemic attack (TIA)


    C.Venous sinus thrombosis (VST)


    D.Complex migraine


  Correct Answer D: Complex migraine. The presence of aura, facial numbness, nausea, and vomiting are typical in a complex migraine. TIA and VST are incorrect in that she is a young woman with no risk factors and she has a normal neuro exam. Bell’s Palsy is a cranial nerve VII palsy, and you would not see the above symptoms with this.


63.A 42-year-old male comes to the emergency department complaining of bilateral lower extremity weakness that began in his feet but has quickly spread proximally up the legs. He is now having difficulty walking. Besides a recent flu-like illness, he is otherwise healthy. All are physical exam findings you could find except:


    A.Hyperreflexia


    B.Lumbar tenderness


    C.Respiratory distress


    D.Facial weakness


  Correct Answer A: Hyperreflexia. Guillain–Barré syndrome usually presents with lower extremity and back pain with rapidly progressing lower extremity weakness. Reflexes are usually decreased or absent. Facial weakness and respiratory distress can occur as the weakness progresses.


64.What diagnostic measure could you do in the emergency department (ED) for the diagnosis of Guillain–Barré syndrome?


    A.MRI of the spine


    B.Lumbar puncture


    C.Vascular ultrasound


    D.Head CT


  Correct Answer B: Lumbar puncture. Cerebrospinal fluid analysis showing elevated protein with normal white blood cell count is indicative of Guillain–Barré and can easily be done in the ED. MRI, vascular ultrasound, and head CT would not confirm the diagnosis and would likely be normal.


65.A 38-year-old female comes to the emergency department complaining of sudden onset of dizziness, described as the room spinning, with nausea and vomiting. Symptoms are worse sitting up and standing. All are signs of a peripheral etiology except:


    A.Resolution of symptoms with Epley maneuver


    B.Horizontal nystagmus


    C.Normal gait


    D.Increased symptoms with Epley maneuver


  Correct Answer D: Increased symptoms with Epley maneuver. The Epley maneuver is done to remove otoconia (otolith) crystals from the semicircular canals that have become dislodged, causing benign paroxysmal positional vertigo. If done correctly, the Epley maneuver would cause resolution of symptoms, not worsening. Horizontal nystagmus and normal gait are both common in peripheral etiologies.


66.A 74-year-old male is brought in by an ambulance after found down by his family. Patient is awake, answering questions, but disoriented to time and place. Medical history: hypertension, coronary artery disease, atrial fib, high cholesterol. Patient is on aspirin, lisinopril, Zocor, Coumadin, hydrochlorothiazide, amlodipine. Vital signs: temp, 97°F; HR, 88; BP, 130/60; RR, 22; and O2, sat 96% on RA. Head CT done in the emergency department confirms a subdural hemorrhage (SDH). What is your next step for management?


    A.Intubation


    B.Arterial line and nicardipine


    C.Vitamin K, fresh frozen plasma, platelets


    D.Keppra


  Correct Answer C: Vitamin K, fresh frozen plasma, platelets. In a patient with a SDH, it is most important to correct anticoagulation. Given that patient is awake, answering questions, and has a normal O2 sat, intubation is not indicated. With a normal BP, nicardipine and arterial line are not indicated. Reversing anticoagulation is more important than starting Keppra to prevent seizures.


67.A 37-year-old male with a history of hepatitis C, intravenous drug use (IVDU), comes to the emergency department complaining of back pain and leg weakness. He states that he cannot walk because of the pain in his back and legs, to the point where he is unable to go to the bathroom. In ED, vital signs are as follows: Temp, 101.4°F; HR, 128; BP, 160/67; RR, 22; and O2 sat 100% on RA. Exam is significant for focal lumbar spine tenderness at L3 and 3/5 strength testing in his lower extremities bilaterally. His most likely diagnosis is:


    A.Epidural abscess


    B.Cauda equina syndrome


    C.Guillain–Barré


    D.Muscle strain


  Correct Answer A: Epidural abscess. Tachycardia, fever, and focal lumbar tenderness, with risk factor of IVDU, suggest an infectious etiology. Cauda equina, though it can cause symptoms of back pain and leg weakness, also causes bowel or bladder dysfunction, and you typically would not see a fever. Guillain–Barré presents with ascending leg weakness. Muscle strain would not typically cause weakness or midline tenderness.


68.A 42-year-old female with history of seizures is brought in by emergency medical services having a seizure. Her family states that she ran out of her Keppra 3 days ago and has now been seizing for about 45 minutes. Patient received 2 mg IV Ativan en route. On exam, vital signs are as follows: Temp, 99.2°F; HR, 110; BP, 140/72; RR, 20; and O2 sat 91% on RA. She is having a tonic-clonic seizure. What is your next step for management?


    A.Ativan drip


    B.Keppra loading dose


    C.Head CT


    D.Intubation


  Correct Answer D: Intubation. This patient is in status epilepticus. Securing the airway is the first step. Ativan drip, Keppra load, and head CT can be done after securing the airway.


69.A 36-year-old female with metastatic breast cancer to lung and bone presents complaining of leg weakness and difficulty ambulating for 1 day. Her exam is notable for 1/5 strength testing in her bilateral lower extremities. What other findings could you expect to find to support your diagnosis?


    A.Hyperreflexia


    B.Normal rectal tone


    C.Bladder scan with 1,500 cc urine


    D.Perineal hyperesthesia


  Correct Answer C: Bladder scan with 1,500 cc urine. The patient described most likely as Cauda equina syndrome from a metastatic lesion. Urinary retention is common in Cauda equina syndrome. You would also expect to find decreased or absent reflexes, decreased rectal tone, and perineal hypoesthesia.


70.A 79-year-old male is brought to the emergency department by family, who state they are having difficulty taking care of him over the last 8 months. He is having memory loss, difficulty with ADLs, and has been wandering out of the house. The family is concerned something is wrong with him. The patient has no complaints and would not have come if it were not for his family. On exam, vital signs are as follows: Temp, 98.5; HR, 86; BP, 123/72; RR, 16; and O2 sat 99% on RA. His physical exam is unremarkable. You do a broad workup, including complete blood count, comprehensive metabolic panel, toxicology screen, urinalysis, chest x-ray, head CT. All of the following support the diagnosis of dementia over delirium except:


    A.Chronic and progressive onset


    B.White blood cell (WBC) 9


    C.Na 119


    D.CT showing white matter changes


  Correct Answer C: Na 119. Metabolic disorders including hyponatremia are a common cause of acute delirium. Dementia is chronic in nature, while delirium is acute and rapid onset. A WBC of 9 is within the normal range, making infection-causing delirium less likely. Chronic white matter changes are typical in an elderly person, and do not represent anything acute.


71.A 28-year-old female, who is G2P1, presents with 2 days of vaginal bleeding with a confirmed urine pregnancy test in the clinic at approximately 8 weeks pregnant from last menstrual period. She reports the bleeding to be light with some spotting and no pain. Vital signs: temp, 98.2°F; HR, 86; BP, 108/58; RR, 16; and O2 sat 100% on RA. On exam: she has a soft, nontender abdomen, and pelvic exam shows trace blood in the vaginal vault, cervical os is closed, no discharge, no adnexal tenderness or cervical motion tenderness. Pelvic ultrasound report: single, intrauterine pregnancy, fetal heart rate present, crown-rump length measuring 1.67 cm. What is the most likely diagnosis?


    A.Complete miscarriage


    B.Threatened miscarriage


    C.Missed miscarriage


    D.Inevitable miscarriage


  Correct Answer B: Threatened miscarriage. Threatened miscarriage is early pregnancy <20 weeks with bleeding and a closed cervical os. There may also be mild pain and cramping. Complete miscarriage occurs when products of conception have passed and is associated with heavy bleeding and cramping, Ultrasound may only see debris (blood) with open os. Missed miscarriage is often detected at the first ultrasound when there is no heartbeat present. No preceding bleeding or pain have occurred. Inevitable Miscarriage is when the cervical os is open (dilated) and bleeding is often heavier, and associated pain and cramping. There is a transition from inevitable miscarriage to complete miscarriage.


72.A 24-year-old female presents to the emergency department with 5 days of left-sided pelvic pain, vaginal discharge, and fevers to 101. She denies urinary symptoms or other infectious symptoms. She was seen in the clinic 5 days ago and treated for pelvic inflammatory disease (PID). Her last menstrual period was 10 days ago. Medications include oral contraceptive and Tylenol. Vital signs: temp, 101.2°F; HR, 114; BP, 105/58; RR, 18; and O2 sat, 97% on RA. Labs demonstrate white blood cell, 14K; red blood cell, 5.15; hematocrit, 38; HG, 12. Urinalysis negative for infection and urine hCG negative. What is the most likely diagnosis?


    A.Ectopic pregnancy


    B.Ovarian cyst


    C.Diverticulitis


    D.Tubo-ovarian abscess


  Correct Answer D: Tubo-ovarian abscess. Complication of PID, usually in reproductive-age females with a history of upper genital infections, can be potentially life-threatening infection with management requiring parenteral antibiotics and often drainage. Ectopic pregnancy risk is increased with history of PID/sexually transmitted infection, but pregnancy test is negative, making ectopic unlikely. Ovarian cysts are fluid-filled sacs that form within the ovaries. They are also noninfectious but can be painful. Diverticulitis can cause similar symptoms with fever and pain, but given her young age, it makes this less likely.


73.What is the recommended first-line treatment for gonorrhea infection?


    A.Ciprofloxacin 500 mg twice a day for 7 days


    B.No treatment recommended


    C.Ceftriaxone 2 mg IV × 1


    D.Ceftriaxone 250 mg intramuscular (IM) × 1


  Correct Answer D: Ceftriaxone 250 mg IM ×1. Centers for Disease Control and Prevention recommends ceftriaxone 250 mg IM ×1 as the initial first-line treatment for gonorrhea for both men and women. Fluoroquinolones are no longer recommended due to high resistance rates. Treatment of gonorrhea is strongly recommended as untreated infection can lead to pelvic inflammatory disease, scarring of the reproductive organs, ascending to the upper pelvic organs and rarely into the bloodstream. Ceftriaxone IV is not recommended for outpatient treatment.


74.A 32-year-old female presents to the emergency department with right-sided pelvic pain that has been worsening over the past 3 days. She describes the pain as waxing and waning with episodes of severe pain that caused her to double over. She denies fevers or urinary symptoms, but has nausea with severe pain episodes. Her last menstrual period was 9 days ago, and she is not currently sexually active. She was unable to get relief with over-the-counter medications and presents to the emergency department for evaluation. Her medical history includes depression, anxiety, endometriosis, and history of ovarian cysts. Surgical history includes exploratory laparotomy for endometriosis, cystectomy. Vital signs: temp, 98.7°F; HR, 122; BP, 135/78; RR, 18; and O2 sat 99% on RA. What is the initial emergency treatment for the suspected diagnosis.


    A.Consult gynecologist (GYN), stat pelvic ultrasound


    B.FAST ultrasound exam for free fluid


    C.Laboratory tests


    D.General surgical consultation for bowel obstruction


  Correct Answer A: Consult gynecologist (GYN), stat pelvic ultrasound. This patient is most likely having ovarian torsion, which is a surgical emergency. She has a history of ovarian cyst as well as past cystectomy. Consulting GYN and obtaining stat ultrasound should not be delayed. Viability of the ovary is time sensitive and any delay for further testing could compromise its viability. Obtaining a fast exam for free fluid is reasonable if concern for ectopic rupture though the patient states she is not sexually active, with a recent menstrual period and vital signs notable only for tachycardia. Do not delay GYN consultation for lab testing with a high clinical suspicion for ovarian torsion. Appropriate workup for bowel obstruction can be obtained only after evaluation for ovarian torsion is negative.


75.A 32-year-old female, G3 P2, approximately 18 weeks pregnant, presents with vaginal bleeding that started today. She had some mild cramping and then the bleeding that has slowed down upon her arrival in the emergency department. She is feeling otherwise well. She reports that she has had prenatal care at another hospital and reports her blood type is O negative. Vital signs: temp, 97.6°F; HR, 86; BP, 108/58; RR, 16; and O2 sat 100% on RA. Exam: Abdomen soft nontender, pelvic exam: small amount of blood in the vaginal vault, os closed, and no active bleeding. There is no cervical motion tenderness or adnexal tenderness. Pelvic ultrasound shows a small sub-chorionic hematoma, fetal heart rate 158, no free fluid. Urinalysis is negative, and urine culture is pending. Complete blood count: white blood cell, 10; red blood cell, 5.1; hematocrit, 34; Hgb, 11. What does the patient need before being discharged?


    A.Counseling on subchorionic hematoma


    B.Treatment for asymptomatic cystitis in a pregnant person


    C.Rhogam


    D.Blood transfusion


  Correct Answer C: Rhogam. Rhogam injection is given to prevent hemolytic disease of the newborn in this pregnancy and any subsequent pregnancy given her Rh screen is negative. Counseling on subchorionic hematoma is also recommended as increased risk of pregnancy loss with increased size of hematoma, increased maternal age, as well as increased gestational age. Urinalysis is negative; so she does not need empiric antibiotics. She has mild anemia in pregnancy, but does not require a transfusion as the hematocrit is stable and she has no further bleeding


76.A 23-year-old female presents with 6 days of vaginal itching and thick, white discharge. She denies fevers. She is not sexually active. She denies urinary symptoms. Vital signs are normal. On exam, there is thick white discharge in the vaginal vault, no cervical motion tenderness, and no adnexal tenderness. pH is 4.0. Wet prep with + hyphae after KOH. What does the patient have?


    A.Bacterial vaginosis


    B.Chlamydia


    C.Candida vulvovaginitis


    D.N. Gonorrhea


  Correct Answer C: Candida vulvovaginitis. The patient has classic symptoms for yeast infection, which include itching, pain, thick white discharge, and wet prep with positive hyphae. pH is usually low/normal for yeast. She has no pain and is not sexually active, but it is recommended to send gonorrhea/Chlamydia test on all patients with vaginal discharge and pain. Bacterial vaginosis can cause itch and discharge, but there is presence of clue cells on wet prep.


77.A 37-year-old healthy female, approximately 35 weeks gestation, is a restrained driver in an automobile accident. She is brought to the ED with symptoms of neck and back pain. Her initial set of vital signs was reported to be normal. While in the ED, she reports sudden onset of abdominal pain and cramping with vaginal bleeding. Vital signs are notable for tachycardia and hypotension. What is the likely diagnosis?


    A.Muscle strain


    B.Abdominal wall contusion


    C.Placental abruption


    D.Threatened miscarriage


  Correct Answer C: Placental abruption. Placental abruption is a condition where the placental lining has separated from the uterus. This separation can be partial or complete. Patients are often tachycardic and hypotensive given blood loss. Muscle strain and abdominal wall contusion may also be present, but given pregnancy and vaginal bleeding, less likely the cause of the abdominal pain. Threatened miscarriage occurs during the early stages of pregnancy, not in third trimester.


78.In the above scenario, what is the next step in the emergency department management of the patient?


    A.Admit to the antepartum floor


    B.Discharge home


    C.Consult obstetric (OB) for stat evaluation and potential emergent C-section


    D.Comprehensive OB ultrasound


  Correct Answer C: Consult OB for stat evaluation and potential emergent C-section. This patient is hemodynamically unstable and may need stat delivery. She will need two large-bore IVs with aggressive fluid resuscitation to maintain perfusion, supplemental O2, possible blood transfusion. If hemodynamic instability is prolonged, then risk of hypoperfusion to the fetus increases. It is recommended for emergent delivery of fetus if mother or fetus becomes unstable. If the patient were stable, consider admission to the antepartum floor. OB ultrasound is a consideration may be delayed or prolong definitive intervention. Recommend bedside focused assessment with sonography for trauma exam to evaluate for bleeding.


79.A 35-year-old female, G1 P0, who is 37 weeks 3 days gestation, presents to the emergency department with headache and abdominal pain. She was not able to get in touch with her obstetrician, and her husband is concerned about her. Her evaluation is notable for BP, 157/78; HR, 103; urine notable for 3+ protein; and her liver function tests are normal. What is the diagnosis?


    A.Urinary tract infection


    B.Preeclampsia


    C.Hypertension in pregnancy


    D.Hemolysis, Elevated Liver enzymes, Low Platelet count (HEELP) syndrome


  Correct Answer B: Preeclampsia. Preeclampsia is defined as elevated blood pressure and proteinuria in pregnancy >20 weeks gestation. UTI is a complication in pregnancy and can contribute to preterm labor though no reported bacteria or urinary symptoms to consider this. Pregnancy-induced hypertension is elevated blood pressure in pregnancy >20 weeks gestation without proteinuria. HELLP is a complication of preeclampsia and is a severe life-threatening condition with constellation of lab abnormalities, including HEELP. Delivery of the fetus is the only known way to treat HELLP.


80.A 63-year-old homeless male with history of daily ETOH ingestion and 20 pack-year smoking history presents with 9 days of productive cough of gelatinous sputum, sweats, and shortness of breath. Vital signs: temp, 101.3°F; HR, 110; BP, 99/60; RR, 28; and O2 sat 90% on RA. Exam reveals crackles in all lung fields, with decreased breath sounds and dullness to percussion in the right upper lobe. Which of the following is most true about his diagnosis?


    A.This patient can be managed outpatient with oral Azithromycin.


    B.Patient likely has community-acquired pneumonia due to Pseudomonas aeruginosa.


    C.Patient’s chest x-ray will show a lobar infiltrate with accompanying pleural effusion, which is the pathognomonic radiographic presentation for pneumonia.


    D.Patient likely has community-acquired pneumonia due to Klebsiella pneumoniae.


  Correct Answer D: Patient likely has community-acquired pneumonia due to K. pneumoniae. K. pneumoniae is the pathogen most likely to cause infection in those with chronic illnesses, including alcohol abuse, and in those with underlying lung disease. P. aeruginosa, on the other hand, is a common pathogen found in hospital-acquired pneumonia. The patient should not be managed as an outpatient since he exhibits hemodynamic instability. There is no pathognomonic radiographic presentation for pneumonia.


81.A 32-year-old, nonverbal female, with history of cerebral palsy and dysphagia with G-tube, presents with her caretaker after 1 month of cough. Patient completed 5 days of Z-pack after diagnosis of pneumonia 2 weeks ago, but continues with cough. According to the caretaker, patient spiked a temp two nights ago. Exam reveals poor oral dentition with gingivitis, cervical lymphadenopathy, and decreased sounds with crackles in lower lobes. Vital signs: temp, 102.0°F; HR, 102; BP, 108/70; RR, 20; and O2 sat 91% on RA. What is the most likely diagnosis?


    A.Lung abscess


    B.Acute bronchitis


    C.Pneumonia that failed outpatient treatment


    D.Pulmonary embolism


  Correct Answer A: Lung abscess. Lung abscesses are caused by pulmonary necrosis and are most likely to occur in the setting of aspiration pneumonia due to the anaerobes present in the gingiva. While bronchitis is characterized by persistent cough, it does not usually have accompanying fever or other constitutional symptoms. Pneumonia that failed outpatient treatment is a possible consideration, but the key features to this case are the history of dysphagia with poor dental hygiene, which predisposes this patient to lung abscess. Pulmonary embolism would be a consideration given that the patient does present with tachycardia and hypoxia. However, it does not fit in this clinical picture. Those with pulmonary emboli will exhibit pleuritic chest pain, dyspnea, hemoptysis, and accentuated pulmonic component to the second heart sound.


82.A 19-year-old female presents with 1-day history of intermittent cough, expiratory wheeze, and tachypnea. The patient is able to answer questions, but can only say three- to four-word sentences. Vital signs: temp, 98.6°F; HR, 100; BP, 155/105; RR, 28; and O2 sat 95% on RA. What is the emergent treatment for this patient?


    A.Metoprolol


    B.Nebulized albuterol sulfate


    C.Get a STAT chest x-ray


    D.IV diphenhydramine


  Correct Answer B: Nebulized albuterol sulfate. β-Adrenergic agonists such as albuterol sulfate are the first-line agents to treat acute bronchospasms from asthma. β-Blockers such as metoprolol, on the other hand, can exacerbate asthma. Chest x-rays are only helpful in the asthmatic setting if there is any concern for infectious causes such as pneumonia. But, in either case, it is not the next step in managing this patient emergently. Antihistamines such as diphenhydramine can reduce the patient’s ability to clear secretions.


83.A very concerned mother brings her 2-month-old son after 2-day history of coughing, fever, rhinorrhea, and “breathing funny.” On exam, you see the child has nasal flaring, accessory muscle use with retractions, and diffuse wheezing on auscultation. Vital signs: 99.9°F, HR, 160; BP, 80/50; RR, 40; and O2 sat 94% on RA. Which of the following is not true?


    A.Respiratory syncytial virus is the most common cause of bronchiolitis.


    B.Heliox should not be considered to manage respiratory distress in infants.


    C.Nebulized epinephrine (1:1,000) should be considered if albuterol fails.


    D.Corticosteroid will be helpful in infants with previous history of reactive airway disease.


  Correct Answer B: Heliox should not be considered to manage respiratory distress in infants. Heliox (helium-oxygen) has been found to be helpful in children with bronchiolitis, or those with postextubation stridor, respiratory distress syndromes, or bronchopulmonary dysplasia.


84.A 63-year-old male presents to the emergency department for the eighth time over 6 months. Patient is complaining of persistent dry cough with each visit. However, with today’s visit, patient complains of worsening shortness of breath over the past week. Patient states he is now having difficulty ambulating to the bathroom without taking a break to catch his breath. On exam, he appears to have slight cyanosis around the mouth, inspiratory crackles, and clubbing. Chest x-ray reveals atelectasis and diffuse fibrosis of the lung. Chest CT reveals diffuse pleural honeycombing. What is the most likely diagnosis?


    A.Congestive heart failure (CHF)


    B.Pulmonary embolism


    C.Idiopathic fibrosing interstitial pneumonia


    D.Pulmonary hypertension


  Correct Answer C: Idiopathic fibrosing interstitial pneumonia. Idiopathic fibrosing interstitial pneumonia given the progressive nature of the symptoms, worsening dyspnea on exertion, and pleural honeycombing on CT is a characteristic of interstitial lung disease. Congestive heart failure will also present with dyspnea on exertion. However, the radiographic findings are not consistent with CHF. Chest x-ray of a CHF patient will likely show cardiomegaly, pleural effusions, or Kerley B lines from interstitial edema. Pulmonary hypertension also presents with progressive exertional dyspnea; however, those with pulmonary hypertension may also complain of exertional angina and often with exertional syncope.


85.A 67-year-old male with longstanding chronic obstructive pulmonary disease (COPD) presents in the ED by emergency medical services with worsening dyspnea over the past 3 hours. Patient was provided with continuous bronchodilator nebulizers en route. Vital signs on arrival to the ED: temp, 97.6°F; HR, 100; BP, 167/98; RR, 26; and O2 sat 90% on 2 L. Patient has diffuse wheezes on exam. You obtain an arterial blood gas, which shows pH 7.25, PCO2 50. Patient is answering questions appropriately but lethargic and only able to answer two to three-word questions at a time. What is the first step in this patient’s management?


    A.Increase the patient’s supplemental oxygen


    B.Provide noninvasive positive pressure ventilation


    C.Chest x-ray


    D.Emergently intubate patient


  Correct Answer B: Provide noninvasive positive pressure ventilation. Providing noninvasive positive pressure ventilation is beneficial for patients with acute exacerbations of COPD with hypercapnic acidosis. Positive results should be seen within 1 to 2 hours. Providing supplemental oxygen to a chronic COPD patient can be harmful. Providing too much oxygen can exacerbate the patient’s hypercapnia. Goal should be to maintain an oxygen level between 88% to 92%. While obtaining a chest x-ray will be important in his emergent care, it is important to stabilize the patient first. Intubation and mechanical ventilation should be the last resort. It should be a consideration if the noninvasive positive pressure ventilation fails or is not tolerated by the patient, or if the patient has respiratory failure.


86.A 21-year-old college student presents with 2 months of progressive cough, shortness of breath, and intermittent chest pain. She reports a 5 to 10 pound weight loss over 2 months with increasing fatigue. Vital signs: temp, 98.6°F; HR, 80; BP, 110/70; RR, 16; and O2 sat 95% on RA. On exam, patient has pupils equal, round, reactive to light, slight swelling to parotid gland, dry oral mucosa, no cervical lymphadenopathy, heart regular rate and rhythm (RRR), lungs exhibit slight expiratory wheeze, soft nontender abdominal, no significant muscle trophy, cranial nerve II-XII grossly intact. Chest x-ray shows bilateral hilar adenopathy and reticular opacities. Which of the following should be excluded from your differential?


    A.Sarcoidosis


    B.Tuberculosis


    C.Pulmonary histoplasmosis


    D.Horner syndrome


  Correct Answer D: Horner syndrome. Horner syndrome does not fit the clinical picture. It is a neurologic syndrome that can occur from a wide variety of reasons, including stroke, injury to brachial plexus, cranial fossa neoplasm, as well as a Pancoast tumor (lung apex). Symptoms include ptosis, anhidrosis, miosis. Sarcoidosis typically presents between ages 20 and 60. Bilateral lymphadenopathy is common with sarcoidosis, while parenchymal findings can vary, but include reticular opacities. Sarcoidosis also causes other constitutional symptoms, including visual changes, dry mouth, parotid swelling, and muscle weakness. Tuberculosis can present with cough, occasional hemoptysis, fever, substernal or interscapular pain, night sweats, anorexia, and weight loss. Exam findings may include posttussive rales. Radiographic findings for tuberculosis include hilar lymphadenopathy, pleural effusions or infiltrates. Pulmonary histoplasmosis should be considered whenever sarcoidosis or tuberculosis are on the differential. Young healthy patients typically have mild symptoms, including chest pain, cough, arthralgias, fever, and rash, including erythema nodosum. Immunocompromised, or elderly patients can also exhibit pericarditis or lung infections. Radiographic findings include pneumonia with mediastinal or perihilar lymphadenopathy, cardiomegaly suggesting pericarditis, pulmonary nodules, or cavitary lung lesions.


87.A 74-year-old male, with a history of diabetes and hypertension, presents with worsening shortness of breath and cough. Patient states that for the past 4 days he has worsening dyspnea on exertion. On exam, decreased breath sounds in the right middle and right lower lobes, and rales on the left lower lobe. Abdomen is soft, nontender. Lower extremities show 1+ pitting edema bilaterally. Vital signs: temp, 98.2°F; HR, 82; BP, 164/78; RR, 22; and O2 sat 94% on 2 L. Chest x-ray shows no cardiomegaly, large, bilateral pleural effusions (R>L). Labs show the following: white blood cell, 10.9; Hgb, 14.2; hematocrit, 41.4; Platelet, 226; Na, 139; K, 4.0; blood urea nitrogen, 14; Cr, 0.6; B-type natriuretic peptide (BNP) is within normal limits. What is the treatment?


    A.Provide more supplemental oxygen


    B.Provide nebulized albuterol


    C.Perform a thoracentesis


    D.Perform pulmonary function testing


  Correct Answer C: Perform a thoracentesis. Thoracentesis is indicated in patients with presence of large pleural effusion without known cause, dyspnea, normal BNP, or clinically obvious heart failure. It will allow for relief of symptoms, as well as fluid examination and improved radiographic visualization of lung parenchyma. Giving additional supplemental oxygen, nebulized bronchodilators, and pulmonary functioning testing is not necessary.


88.A 35-year-old schoolteacher presents with persistent dry cough for the past 4 weeks. Patient states that the illness started with general malaise, increased lacrimation, rhinorrhea, and injected conjunctiva. While these symptoms resolved after a few days, her cough persists. She states the cough is paroxysmal in nature and can occur during the day or night. During your exam, lungs are clear with inspiration, but during expiration, the patient has multiple episodes of vigorous coughing, resulting in vomiting. Vital signs are within normal limits. What is the likely diagnosis?


    A.Viral upper respiratory infection


    B.Acute bronchitis


    C.Community-acquired pneumonia (CAP)


    D.Pertussis


  Correct Answer D: Pertussis. Pertussis is a contagious respiratory illness that causes persistent cough with one or more of the classical symptoms: inspiratory whoop, paroxysmal cough, and posttussive emesis and best describes how the patient presented. Upper respiratory infections are viral infections that affect the nasal passages, pharynx, larynx, or sinuses. Symptoms include nasal congestion, cough with or without sputum production, low-grade temp, and postnasal drip. Illness is self-limited and usually resolves after 2 weeks. Acute bronchitis is a viral infection resulting in inflammation of the trachea, bronchi, and bronchioles. Symptoms include cough with or without sputum production, dyspnea, general malaise, sore throat, and expiratory rhonchi or wheeze. For healthy patients, symptoms resolve after 2 weeks. Pneumonia results from the inflammation and infection of the alveoli and interstitium of the lung. Symptoms include productive cough, dyspnea, fever, sweats, and rigors. Symptoms for bacterial CAP will continue to worsen until managed with antibiotics.


89.A healthy 28-year-old female who takes only oral contraceptive pills presents with 5 days of nonproductive cough, sore throat, headache, and arthralgias. On exam, head, eyes, ears, nose, and throat are unremarkable, no cervical lymphadenopathy, heart is regular rate and rhythm (RRR), lungs exhibit slight wheeze. Vital signs: temp, 99.1°F; HR, 82; BP, 120/70; RR, 16; and O2 sat 98% on RA. How should you manage this patient?


    A.Ceftin


    B.Supportive treatment, including hydration, cough suppressants, and β-2 agonists


    C.Azithromycin


    D.Bactrim


  Correct Answer B: Supportive treatments, including hydration, cough suppressants, and β-2 agonists. Acute bronchitis is a viral illness of the trachea, bronchi, and bronchioles. In healthy individuals, supportive measures are the proper treatment. Antibiotics are used in those with acute exacerbations of chronic bronchitis, the elderly, those with cardiopulmonary diseases or who are immunocompromised, or those with cough for more than 7 to 10 days.


90.A 27-year-old male, with a known history of a shellfish allergy, presents to your emergency room by ambulance after accidently eating a shrimp-based sauce. The emergency medical technician reports he was found to have lip swelling, wheezing, and complaints of feeling his throat closing. They quickly administered epinephrine and were able to bring him to your emergency department within 10 minutes. He is still symptomatic; however, he is able to speak to you and his vital signs (VS) are normal. What is the next appropriate treatment for this patient?


    A.0.3 mg epinephrine 1/1,000 intramuscular (IM) q10 to 15 minutes, if still symptomatic, consider epinephrine drip


    B.0.3 mg epinephrine 1/100 IM × 1, then start an epinephrine drip


    C.0.3 mg epinephrine 1/1,000 subcutaneous (SC) q10 to 15 minutes, if still symptomatic, consider epinephrine drip


    D.0.3 mg epinephrine 1/100 SC q10 to 15 minutes, if still symptomatic, consider epinephrine drip


  Correct Answer A: 0.3 mg epinephrine 1/1,000 intramuscular (IM) q10 to 15 minutes, if still symptomatic, consider epinephrine drip. Studies have shown that in a state of anaphylaxis, intramuscular injection of epinephrine is more rapidly absorbed than other routes. This patient does not necessarily need to be started on a drip; this decision is also based on his clinical appearance and VS. Therefore, you should consider only after re-evaluating the patient after IM injections.


91.A 66-year-old male, noninsulin-dependent diabetic, presents to his primary care doctor complaining of a pruritic insect bite to his right leg that has now become red and warm. He also admits to feeling subjective fevers and fatigue. His vital signs are normal, and on exam he has a warm, erythematous 5 × 6 cm area to his lower leg that is painful. What is recommended for the initial treatment of this patient?


    A.Start hydrocortisone ointment and take nonsteroidal anti-inflammatory drugs as needed for pain


    B.Start cephalexin 500 mg qid, advised elevate and f/u in 48 for reevaluation


    C.Admit for IV Ancef, dermatology consult, and leg immobilization


    D.Admit for IV vancomycin, leg elevation, and monitoring


  Correct Answer B: Start cephalexin and reevaluate in 2 days. This patient has cellulitis, and although describing, some systemic symptoms have normal vital signs. It is recommended to treat for staph/strep and should follow up to reevaluate if symptoms have improved. If they have not, it may be warranted to admit for more broad-spectrum and parental antibiotics.


92.A 15-year-old male presents to your emergency department with his mother complaining of an itchy rash on his back. Otherwise she has admitted that he is in a normal state of health and recently started playing soccer. On exam you see a circular erythematous raised border with central clearing. Skin looks dry. What is the next appropriate treatment?


    A.Start Keflex 500 mg po qid × 7 days


    B.Start doxycycline 100 mg po bid × 21 days


    C.Start clotrimazole 1% topically × 14 days


    D.Start fluconazole 100 mg po qd × 14 days


  Correct Answer C: Start clotrimazole 1% topically 3 14 days. This topical antifungal is used in cases of suspected tinea corporis or ringworm. This is an immunocompetent patient with a probably fungal infection especially after starting soccer. There is no indication to start oral antifungals. Also there are no signs or symptoms of a bacterial infection or cutaneous manifestations of Lyme.


93.A 46-year-old female nurse comes to your emergency department after a possible blood-borne exposure while at work today. She was wearing gloves while placing an IV; she accidently stuck herself with the 22-gauge needle. She immediately took off her gloves and saw a tiny puncture wound with minimal bleeding. She washed her wound copiously and states she is up-to-date with all her vaccines, but the source is high risk. What is the most appropriate treatment?


    A.Raltegravir


    B.Truvada


    C.Tenofovir-emtricitabine and Raltegravir


    D.Does not meet criteria for postexposure prophylaxis treatment at this time


  Correct Answer C: Tenofovir-emtricitabine and Raltegravir. Truvada is the coformulated name for Tenofovir-emtricitabine, which is non-nucleoside reverse transcriptase inhibitors. These work best with either an integrase inhibitor.


94.A patient presents to your emergency department within 72 hours of being possibly exposed to HIV. Postexposure prophylactic treatment (PEP) for HIV is recommended in which case(s).


    A.Repetitive vaginal intercourse with a high-risk person


    B.Reparative anal intercourse with a high-risk male


    C.Repetitive oral sex with a high-risk female


    D.All of the above


  Correct Answer D: All of the above. It may be appropriate to recommend PEP for anyone exposed sexually to a high-risk person within 72 hours. Based on the ability to test, the source and the duration of treatment may vary.


95.A 39-year-old laboratory technician reports to the emergency department after spilling a urine sample over her hands. She states the patient has HIV, hepatitis C, and high-risk behaviors. On exam, there are no lacerations or skin disruption visible. What is the next most appropriate treatment?


    A.Wash area with soap and water, start postexposure treatment for HIV/hepatitis C, administer Tdap, follow-up with occupational health


    B.Start postexposure treatment for HIV/hepatitis C, obtain rapid serum HIV test


    C.Wash area with soap and water, start oral antibiotics, and follow-up with occupational health


    D.Wash the affected areas with soap and water


  Correct Answer D: Wash the affected areas with soap and water. The Centers for Disease Control and Prevention does not recommend postexposure prophylaxis (PEP) in the following conditions: after 72 hours of exposure or when there is no visible blood to fluids (urine, nasal secretions, saliva, sweat, or tears). It does recommend PEP for exposures to someone’s genitals, eyes, mouth, or mucous membranes, nonintact skin, or percutaneous exposures. There is no indication for antibiotics at this time.


96.A 65-year-old male with history asthma presents with a rash for 1 year. Patient has several areas of dry, red, and itchy skin mostly located on his face, neck, and also to the insides of his elbows and knees. What is the most likely diagnosis?


    A.Psoriasis


    B.Eczema


    C.Rosacea


    D.Contact dermatitis


  Correct Answer B: Eczema. Psoriasis presents more as erythematous plagues with silvery, white scales, and does not necessarily need to be on the insides of joints. Rosacea is usually on the face. Contact dermatitis affects only the area of the body that was in contact with a particular allergen; it is usually associated with pustules, sometimes linear.


97.A 23-year-old female presents to the emergency department complaining of joint pain, swelling over the last 2 months. She also feels fatigue, poor appetite, and has recently developed an erythematous rash on the cheeks of her face. She just spent a week vacationing at the beach and feels her rash is now worse. What is the most likely diagnosis?


    A.Rosacea


    B.Systemic lupus erythematosus


    C.Eczema


    D.Melasma


  Correct Answer B: Systemic lupus erythematosus. Although rosacea, melasma, and eczema may present with similar rashes, we would not expect to have any joint pain, swelling, fatigue with this etiologies. The above are some of the more common presentations of Lupus; however, they are not limited.


98.An 80-year-old female with history of hypertension and osteoarthritis presents to the emergency department with a painful rash × 2 days. Patient admits that she had just recovered from pneumonia and finished taking Levaquin. Her rash is on the left side of her back; she describes that her rash is extremely painful and sensitive to touch. On exam she has multiple erythematous lesions, some with vesicles, some with scabbing, and very tender to touch. Which is the most appropriate treatment for her diagnosis?


    A.Valacyclovir 1,000 mg po q8h × 7 days


    B.Acyclovir 800 mg po q8h × 7 days


    C.Acyclovir 500 mg po 5×/day × 7 days


    D.Valacyclovir 1,000 mg q12h × 7 days


  Correct Answer A: Valacyclovir 1,000 mg po q8h 3 7 days. Acyclovir would be an appropriate treatment for herpes zoster; however, it is dosed at 800 mg po 5×/day × 7 days. Antiviral treatment has been shown to have benefit when administered within 72 hours of symptom onset. Prednisone is not always recommended.


99.An 18-year-old female in otherwise good health reports to the emergency department after a syncopal episode following a long run. She complains of feeling lightheaded and itchiness all over. She has obvious swelling to her hands and feet. Her blood pressure is 80/40, and pulse is 148. Lung exam reveals diffuse wheezing. Her blood sugar is 102 mg per dL. The most important initial IV therapy would be:


    A.Epinephrine


    B.Diphenhydramine HCL


    C.Methylprednisolone succinate


    D.Normal saline


  Correct Answer A: Epinephrine. The patient presents with exercise-induced anaphylaxis. With hypotension and tachycardia, the patient’s most important initial therapy is epinephrine. Epinephrine has an α-1 adrenergic response that has vasoconstrictor effects, which decreases mucosal edema, elevates blood pressure, and prevents and relieves shock.


100.A young man presents to the emergency room after eating sushi 30 minutes prior with facial flushing, headache, abdominal pain, nausea, vomiting, diarrhea, and palpitations. What is the most likely diagnosis, and what is the treatment?


    A.Campylobacter—IV fluids and azithromycin


    B.Clostridium difficile—IV fluids, metronidazole, and antiemetics


    C.Scombroid—IV fluids, diphenhydramine


    D.Staph—IV fluids, loperamide


  Correct Answer C: Scombroid—IV fluids, diphenhydramine. Scombroid usually results from ingesting Tuna and Mackerel. It causes histamine intoxication. It is best treated with IV fluids and Benadryl. Campylobacter usually presents 3 to 5 days after ingestion from undercooked meat or contaminated water. C. difficile usually results from recent antibiotic use. Staphylococcus usually presents 1 to 6 hours after ingestion.


101.An 80-year-old male on Coumadin for a pulmonary embolism presents to emergency department status post a mechanical fall 4 hours ago. He fell hitting his face. He has multiple abrasions to the side of the face. There was no reported loss of consciousness. His head CT was negative. His international normalized ratio (INR) is elevated to 6.0. What is the proper management that should be initiated in the emergency room?


    A.Omit warfarin altogether


    B.Administer 1 mg vitamin K orally and check INR in 24 hours


    C.Omit one to two doses of warfarin and administer fresh frozen plasma (FFP); recheck INR in 24 hours


    D.Omit one to two doses of warfarin, administer 1 mg vitamin K orally, and recheck INR in 24 hours


  Correct Answer D: Omit one to two doses of warfarin, administer 1 mg vitamin K orally, and recheck INR in 24 hours. An INR >5 and <9 without signs of bleeding include omitting one or two doses of warfarin and more frequent INR checks. Oral vitamin K is indicated. FFP is not indicated without any life-threatening bleeding.


102.A 14-year-old girl suffers from severe menorrhagia since menarche at the age of 12. Her Hgb is 6.9 (normal 12 to 16 g per dL). Her labs indicate a hypochromic microcytic anemia. Her pregnancy test is negative. The most common coagulopathy associated in adolescents with menorrhagia is:


    A.Hemophilia A


    B.Hemophilia B


    C.Von Willebrand


    D.Factor V Leiden


  Correct Answer C: Von Willebrand. Von Willebrand is the most common hereditary disorder. It represents 1%. It is the leading coagulopathy in adolescent menorrhagia. The treatment includes intranasal or IV desmopressin acetate if mild or von Willebrand factor concentrate if severe. Oral contraceptive pill may provide control. Hemophilia is rare in girls. Factor V Leiden is a disorder that increases the risk of clots.


103.A 63-year-old female presents with a fever, anemia, renal failure, and mental status changes. On exam you note fever to 101, petechiae on legs. Labs reveal a differential that notes schistocytes. What is your likely diagnosis?


    A.Idiopathic thrombocytopenic purpura (ITP)


    B.Thrombotic thrombocytopenic purpura (TTP)


    C.Meningitis


    D.Endocarditis


  Correct Answer B: Thrombotic thrombocytopenic purpura (TTP). For TTP just remember FATRN. The pneumonic represents fever, hemolytic anemia, thrombocytopenia, renal failure, and neurologic dysfunction. It is a condition where blood clots form in the small blood vessels throughout. The red blood cells break apart faster than the body can produce them. Treatment involves fresh frozen plasma and plasma exchange. ITP does not present with anemia. The morphology of red blood cells and leukocytes are normal. Meningitis and endocarditis are less likely to cause all of the symptoms in her presentation.


104.A 58-year-old male with history of hypertension on lisinopril for 5 years presents with sudden onset of swelling of the tongue and uvula. On exam, vital signs remain stable. His O2 level is 98% on room air. His respiratory rate is 18. He appears in no distress. He is not drooling. The most appropriate management for this patient is:


    A.Immediate intubation


    B.IV methylprednisolone, IV fluids, O2, and observation for 1 hour


    C.Epinephrine subcutaneous, IV Benadryl, IV fluids, and O2, and observation for 12 to 24 hours


    D.Discharge with antihistamines and steroid taper


  Correct Answer C: Epinephrine subcutaneous, IV Benadryl, IV fluids, O2, and observation for 12 to 24 hours. The patient needs observation of his airway for at least 12 hours with ace inhibitor angioedema. He is stable with no distress; therefore, intubation is not warranted. He should not be discharged with airway swelling.


105.A 46-year-old female undergoing outpatient oral chemotherapy for metastatic ovarian cancer presents with 24 hours of fever, diarrhea, and fatigue. She has no indwelling catheters. Her fever is 101.3; BP is 128/58; heart rate is 92; O2 sat is 98% on room air. Labs reveal an absolute neutrophil count of <500 cells per µL. Hepatic and renal functions are normal. Chest x-ray is normal. Urinalysis is normal. The best management of this patient initially is:


    A.IV moxifloxacin × 1 dose, and discharge on Augmentin and ciprofloxacin orally with close follow-up


    B.IV cefepime 2 g IV q8h and oncology admission


    C.IV vancomycin (15 mg per kg) IV q12h and oncology admission


    D.IV Unasyn 3 g IV and discharge on Augmentin and ciprofloxacin


  Correct Answer B: IV cefepime 2 g IV q8h and oncology admission. Initiation of monotherapy with an antipseudomonal β-lactam agent has been shown to be effective. Other agents such as vancomycin can be added with complicated presentations such as hypotension, pneumonia, indwelling catheters, or significant history of Methicillin-resistant Staphylococcus aureus. Fluoroquinolones is not the first line. Unasyn is not recommended, as it is not a broad-spectrum antibiotic.


106.A 57-year-old male presents to the emergency department status post discharge from the hospital yesterday following an admission for a lower gastrointestinal (GI) bleed. During his stay over the past 5 days, he had a hematocrit as low as 19. He was transfused two units of packed red blood cells upon admission and went to the GI service for a colonoscopy, where the bleeding was stopped. He presents to the emergency room today with possible transfusion reaction. The most common symptom present is:


    A.Fever and chills


    B.Urticaria


    C.Thrombocytopenia


    D.Wheezing


  Correct Answer A: Fever and chills. The most common symptom of transfusion reaction is fever and chills. This can occur even when the blood is correctly matched. Urticaria and wheezing can occur, but less commonly than fever and chills. Hemolytic reactions are rare.


107.An 8-week pregnant, G1P0AB1, female presents with heavy vaginal bleeding, which began today. She has had no prenatal care thus so far. Upon exam, her vital signs are normal. There are clots in the vaginal vault, and her cervical os is open. Pelvic ultrasound reveals no intrauterine pregnancy. Her blood type reveals that she is Rh incompatible. It means that her blood type is:


    A.Rh positive


    B.Rh negative


  Correct Answer B: Rh negative. Rh incompatibility is a condition that develops when a pregnant woman has Rh-negative blood and the baby in her womb has Rh-positive blood. Rhogam should be given to an Rh-negative woman during every pregnancy, during miscarriage or abortion, after prenatal tests such as amniocentesis and chorionic villus biopsy, after injury to the abdomen during pregnancy.


108.A 19-year-old male with hemophilia A presents to the emergency room with severe pain and swelling to his left knee after a mechanical fall while walking down the stairs. On exam, his vital signs are normal. He is in moderate pain. His left knee has an obvious effusion. The best management is:


    A.Factor VII replacement, arthrocentesis, and pain control


    B.Factor VII replacement and pain control


    C.Factor VIII replacement, arthrocentesis, and pain control


    D.Factor VIII replacement and pain control


  Correct Answer D: Factor VIII replacement and pain control. Factor VIII is the correct concentrate. Hemarthrosis is considered a major hemorrhage in a hemophiliac. Arthrocentesis should be avoided unless pain is severe and synovial tension is present.


109.A 55-year-old male with history of hypertension, bipolar disorder on metoprolol, simvastatin, and lithium presents to the emergency room complaining of 3 months of progressive polydipsia and polyuria. Basic metabolic panel demonstrates a Na of 148, a creatinine of 2.2, and a serum glucose of 108. Urinalysis is negative for leukocyte esterase, nitrites, or blood, with a specific gravity of 1.005. The most likely diagnosis is:


    A.Acute lower urinary tract infection


    B.Obstructive uropathy


    C.Diabetic nephropathy


    D.Lithium-induced diabetes insipidus


  Correct Answer D: Lithium-induced diabetes insipidus. Chronic lithium ingestion interferes with the normal response to antidiuretic hormone, leading to a decrease in the ability of the kidney to concentrate the urine by removing free water. This causes excessive thirst and excretion of large volumes of dilute urine. Diabetic nephropathy is unlikely given a normal serum glucose. A negative urinalysis eliminates acute urinary tract infection. Obstructive uropathy is unlikely in the absence of pain and microscopic hematuria.


110.Salicylate toxicity is characterized by the following acid-base abnormalities?


    A.Primary respiratory alkalosis plus a primary metabolic acidosis


    B.Respiratory alkalosis


    C.Metabolic acidosis


    D.Metabolic alkalosis


  Correct Answer A: Primary respiratory alkalosis plus a primary metabolic acidosis. Salicylate toxicity is marked by an initial primary respiratory alkalosis (due to direct stimulation of respiratory centers), followed by a metabolic acidosis (mainly attributable to production of lactic acid), resulting in a mixed acid-base disorder.


111.A 26-year-old male presents to the emergency department with altered mental status after running a marathon. Exam is notable for temp 98.8°F, HR 110, BP 118/62, RR 26, O2 sat 98% on RA without focal neurologic deficits. Basic metabolic panel demonstrates Na of 117. Which is the next step in management?


    A.Immerse the patient in cold water


    B.IV normal saline (NS) infusion


    C.IV 3% saline infusion


    D.IV D5 NS infusion


  Correct Answer C: IV 3% saline infusion. In severely symptomatic patients with hyponatremia, hypertonic (3% saline) is first-line treatment. Normal saline can exacerbate hyponatremia in patients with syndrome of inappropriate antidiuretic hormone secretion. Dextrose solutions are not useful in treating hyponatremia. Cooling techniques are used in acute heat stroke, but are not helpful for hyponatremia.


112.A 76-year-old female recently treated with high-dose prednisone for polymyalgia rheumatica presents to the emergency department complaining of nausea, vomiting, and confusion, with 4/4 systemic inflammatory response syndrome criteria and systolic blood pressures in the mid-80s refractory to fluid resuscitation. Aside from appropriate antibiotics for presumed sepsis, the most important treatment is which of the following?


    A.Additional IV normal saline bolus


    B.IV norepinephrine infusion via central access


    C.100 mg IV hydrocortisone


    D.50 mEq IV sodium bicarbonate


  Correct Answer C: 100 mg IV hydrocortisone. This patient’s refractory hypotension is likely due to adrenal crisis. The most important treatment in this case is stress dose hydrocortisone. Additional IV fluids and pressors, while important, will not work in the absence of glucocorticoids. While sodium bicarbonate may have a role in treating concomitant hyperkalemia, it is not the best answer here.


113.The most common metabolic abnormalities associated with Addison disease include which of the following?


    A.Hypernatremia/hypokalemia


    B.Hyponatremia/hyperkalemia


    C.Hyponatremia/hyperglycemia


    D.Hypernatremia/hypocalcemia


  Correct Answer B: Hyponatremia/hyperkalemia. Addison disease is a chronic endocrine disorder in which the adrenal glands do not produce adequate glucocorticoids (and at times, mineralocorticoids). These patients develop hyponatremia due to the kidneys’ inability to excrete free water in the absence of cortisol. Hyperkalemia is also common due to insufficient aldosterone production.


114.What is the most common cause of hypoparathyroidism?


    A.Hemochromatosis


    B.Hypothyroidism


    C.Autoimmune disease


    D.Surgical procedures of the neck/throat


  Correct Answer D: Surgical procedures of the neck/throat. Surgical procedures of the neck remain the most common cause of acquired hypoparathyroidism. Although with improvements in surgical techniques, autoimmune processes may overtake iatrogenic causes in the near future. There is no causal relationship between hypothyroidism and disorders of the parathyroid. Hemochromatosis can lead to parathyroid dysfunction, but is not the most common cause.


115.A 47-year-old female, post-op day two from thyroidectomy, presents to the emergency department complaining of fatigue, muscle cramps, as well as paresthesias of her fingertips and perioral area. Which will be most helpful in establishing a diagnosis?


    A.Comprehensive metabolic panel


    B.Noncontrast head CT


    C.Electrocardiogram


    D.Chest x-ray


  Correct Answer A: Comprehensive metabolic panel. This patient’s symptoms appear to be consistent with hypocalcemia, which is most easily diagnosed with laboratory studies. In addition to serum Ca, measurement of serum albumin is paramount to distinguish true hypocalcemia from factious hypocalcemia. Computed tomography scans, ECG, and chest x-ray may otherwise be indicated in this patient’s workup, but will not be the most helpful of the diagnostic tools listed.


116.Which of the following is a life-threatening complication of treating hyponatremia?


    A.Acute coronary syndrome


    B.Central pontine myelinolysis


    C.Subarachnoid hemorrhage


    D.Diabetes insipidus


  Correct Answer B: Central pontine myelinolysis. Correcting serum Na too rapidly can precipitate central pontine myelinosis, a neurologic complication that can produce spastic quadriparesis, swallowing dysfunction, and mutism. The rate of correction should not exceed 8 to 12 mEq/L/day. Subarachnoid hemorrhage (SAH) is associated with cerebral salt wasting, which is a potential cause of hyponatremia. However, SAH is not a complication associated with hyponatremia treatments. Acute coronary syndrome and diabetes insipidus are not known complications of hyponatremia treatments.


117.Which of the following agents most rapidly counteracts the cardiac effects of hyperkalemia?


    A.Albuterol


    B.Calcium gluconate


    C.Kayexalate


    D.Insulin-dextrose


  Correct Answer B: Calcium gluconate. Calcium gluconate counteracts the cardiac effects of hyperkalemia by stabilizing the cardiac cell membrane against undesirable depolarization. Insulin administered with dextrose acts to facilitate the uptake of glucose into cells, bringing K with it. Albuterol also facilitates the shift of K intracellularly. Kayexalate binds K in the gut, with an onset of action of 2 to 24 hours.


118.Pseudohyponatremia is caused by which of the following?


    A.Diarrhea


    B.Marked hyperlipidemia/hypertriglyceridemia


    C.Hypocalcemia


    D.Use of loop diuretics


  Correct Answer B: Marked hyperlipidemia/hypertriglyceridemia. Extraordinarily high blood levels of lipid can interfere with certain laboratory tests of serum Na concentration, resulting in an erroneously low Na measurement. Loop diuretics are sometimes used with intravenous normal saline to treat hyponatremia. Large-volume diarrhea may cause hypovolemic hyponatremia. Hypocalcemia is not associated with pseudohyponatremia.


119.A 60-year-old male, with a history of prostate cancer, presents to the emergency department with atraumatic low back pain for 2 weeks with new left lower extremity weakness and urinary incontinence. What is the first thing you would do for this patient?


    A.Order IV steroids


    B.Perform a postvoid residual urine volume


    C.Order lumbar spine x-rays


    D.Order an emergent cord compression protocol MRI


  Correct Answer D: Order an emergent cord compression protocol MRI. The patient’s symptoms are concerning for cord compression which he is at risk for with his prostate cancer; so emergency MRI is necessary. A postvoid residual volume would be helpful in this patient to determine whether he is retaining urine, but is not the first thing that should be done in his management. Lumbar x-rays would not be helpful in this diagnosis as they will not be able to assess the spinal cord. While IV steroids may be part of the treatment plan, confirming the diagnosis of cord compression as soon as possible is most important.


120.A 65-year-old male, with history of hypertension (HTN) and gout, presents with atraumatic left great toe pain, erythema, and warmth for 3 days, similar to his gout flares in the past. He takes lisinopril, metoprolol, hydrochlorothiazide, and hydralazine for his HTN. What mediation would you recommend so that he stops taking to prevent future flares?


    A.Hydrochlorothiazide (HCTZ)


    B.Lisinopril


    C.Metoprolol


    D.Hydralazine


  Correct Answer A: Hydrochlorothiazide (HCTZ). HCTZ can precipitate acute gout flares or hyperuricemia in patients with a history of gout, a familial predisposition to gout, or chronic renal failure. The other antihypertensive medications do not have this adverse reaction.


121.What test would you perform to further confirm that a patient has carpel tunnel syndrome?


    A.Adson test


    B.Thompson test


    C.Phalen test


    D.Empty can test


  Correct Answer C: Phalen test. Phalen test as well as Tinel test can help confirm the diagnosis of carpel tunnel. Phalen test is performed by having the patient fully flex the palms at the wrist with the elbow in full extension to provide extra pressure on the median nerve. Alternatively, the backs of the hands are placed against each other to provide hyperflexion of the wrist and the elbows remain flexed. A positive Phalen sign is defined as pain and/or paresthesias in the median-innervated fingers, with 1 minute of wrist flexion. Adson test is performed to assist in the diagnosis of thoracic outlet syndrome, while the Thompson test helps facilitate the diagnosis over Achilles rupture. Empty can test evaluates the strength and integrity of the supraspinatus muscle and tendon for rotator cuff injury.


122.A 35-year-old female presents with atraumatic left foot pain for 3 weeks. The pain is primarily over the bottom of her foot and is worse when she has to get up to go to the bathroom in the middle of the night and the morning. What is your working diagnosis?


    A.Heel spur


    B.Plantar fasciitis


    C.Peripheral neuropathy


    D.Osteoarthritis


  Correct Answer B: Plantar fasciitis. Many patients with plantar fasciitis have the “first step in the morning” symptom, in which the pain occurs for the first few minutes after getting out of bed and then resolves, but as the disease progresses, the pain can continue throughout the day. While a heel spur can cause heel pain, there is no direct relationship to heel spurs and plantar fasciitis. It would be unusual for a 35-year-old to have osteoarthritis as well as neuropathy without diabetes, and this is an atypical presentation of both.


123.A 25-year-old male with a history of intravenous drug use presents to the emergency department with atraumatic low back pain, and there is concern for osteomyelitis. What lab studies would be helpful in the diagnosis?


    A.Wound culture


    B.Erythrocyte sedimentation rate (ESR)


    C.Lactate dehydrogenase (LDH)


    D.Rheumatoid factor


  Correct Answer B: Erythrocyte sedimentation rate (ESR). ESR is nonspecific; however, it is elevated in 90% of cases. Superficial wound or sinus tract cultures often do not correlate with the bacteria that are causing osteomyelitis and have limited use. LDH is useful in the diagnosis of red blood cell hemolysis, is elevated in nonspecific tissue damage and liver damage, and is also one of the factors correlating with reduced survival in patients with non-Hodgkin lymphoma. Rheumatoid factor is used as a marker for patient with suspected rheumatoid arthritis or other immunologic conditions.


124.What would you expect to see on joint aspiration in a patient with pseudogout?


    A.Uric acid crystals


    B.Calcium pyrophosphate crystals


    C.Calcium oxalate


    D.Calcium phosphate


  Correct Answer B: Calcium pyrophosphate crystals. Calcium pyrophosphate is found in synovial fluid in pseudogout. Uric acid crystals are found in gout. About 70% of kidney stones are made of calcium oxalate, and about 10% are made of calcium phosphate.


125.A 35-year-old female who works as a house cleaner presents to the emergency department with atraumatic right knee pain for 2 weeks with swelling, but she continues to have full range of motion. What is your working diagnosis?


    A.Osteoarthritis


    B.Prepatellar bursitis


    C.Iliotibial band syndrome


    D.Septic arthritis


  Correct Answer B: Prepatellar bursitis. Prepatellar bursitis, also known as “housemaid’s knee,” “coal miner’s knee,” “carpet layer’s knee,” is inflammation of the prepatellar bursa and can be caused by an acute traumatic event or repeated trauma, such as one kneeling daily. Iliotibial band syndrome causes lateral knee pain typically in runners or cyclers. Osteoarthritis would be less likely in a 35-year-old, and septic arthritis would be less likely with full range of motion.


126.A 10-year-old boy presents to the emergency department with atraumatic right knee pain for 1 week with a lump. What is your working diagnosis?


    A.Osgood Schlatter disease


    B.Avascular necrosis


    C.Legg-Calvé–Perthes disease


    D.Hemarthrosis


  Correct Answer A: Osgood Schlatter disease. Osgood Schlatter disease is an inflammation of the patellar ligament at the tibial tuberosity, which presents as painful lumps below the knee and most common in young adolescents during skeletal development, typically aggravated by running and jumping. Avascular necrosis usually involves the epiphysis of long bones, such as the femoral and humeral heads, and most commonly affects the hip and the fourth and fifth decade of life. Legg–Calvé–Perthes disease is avascular necrosis of the proximal femoral head resulting from compromise of the blood supply to this area, usually occurring in children aged 4 to 10 years. It has an insidious onset and may occur after an injury to the hip. Hemarthrosis typically occurs in a patient who has a blood disorder such as hemophilia or someone who is anticoagulated.


127.What medication is not indicated for the acute treatment of gout?


    A.Indomethacin


    B.Colchicine


    C.Allopurinol


    D.Prednisone


  Correct Answer C: Allopurinol. All of these medications are indicated in the acute treatment of gouty arthritis except for allopurinol as it can cause an acute attack or worsen the existing attack.


128.What blood test is not helpful in the diagnosis of rheumatoid arthritis?


    A.RF (rheumatoid factor)


    B.ANA (antinuclear antibody)


    C.Erythrocyte sedimentation rate


    D.Lactate dehydrogenase (LDH)


  Correct Answer D: Lactate dehydrogenase (LDH). All of these tests are helpful except LDH. LDH is useful in the diagnosis of red blood cell hemolysis, is elevated in nonspecific tissue damage and liver damage, and is also one of the factors correlating with reduced survival in patients with non-Hodgkin lymphoma.


129.A 15-year-old boy is brought to the emergency department by his mother with atraumatic worsening right hip pain over the past 2 weeks. He denies fever/chills. Labs show white blood cell (WBC) 5k; x-ray shows distal femoral lytic lesion with “onion-skin” appearance. Which of the following is the most likely diagnosis?


    A.Slipped capital femoral epiphysis


    B.Legg–Calvé–Perthes disease


    C.Ewing sarcoma


    D.Septic arthritis


    E.Osteomyelitis


  Correct Answer C: Ewing sarcoma. Ewing sarcoma is correct given the “onion-skin” appearance of periosteal reaction. Ewing sarcoma is a rare malignant bone tumor seen in teenagers and young adults, typically found in the femur or pelvis. Osteomyelitis and septic arthritis can present similarly, but less likely given absence of fever or elevated WBC. Slipped capital femoral epiphysis would show displacement of the femoral head on x-ray, and Legg–Calvé–Perthes disease would show sclerosis of the femoral head, indicating avascular necrosis.


130.A 45-year-old male with a history of noninsulin-dependent diabetes mellitus and peptic ulcer disease presents with an atraumatic right knee pain and swelling for 18 hours associated with erythema, warmth, and painful ROM. Denies fever or chills. You perform arthrocentesis, and synovial fluid is cloudy, yellow, white blood cell (WBC) 35K, with needle-shaped crystals. What is the best initial treatment option?


    A.IV antibiotics


    B.Colchicine


    C.Oral corticosteroids


    D.Indomethacin


    E.Opiates


  Correct Answer B: Colchicine. The patient has gout given the needle-shaped crystals and findings consistent with inflammatory synovial fluid. Antibiotics would be indicated in septic arthritis, which is diagnosed when fluid has WBC >50k without crystals. Typically, first-line therapy would be NSAIDs such as indomethacin, but in this case, it is contraindicated given history of peptic ulcer disease causing concern for bleeding risk. Opiates are added only for breakthrough pain, not initial therapy. Oral corticosteroids may be used to treat gout, but should be reserved for inability to take or failure of nonsteroidal anti-inflammatory drugs and colchicine. Colchicine is most effective when administered within the first 24 hours of symptom onset.


131.A 22-year-old female, who delivered a healthy baby by C-section 1 day ago, presents to the emergency department with lower abdominal pain. Her vitals are as follows: temp, 102°F; HR, 110; BP, 130/70; RR, 20; and O2 Sat 100% on RA. On exam, she has lower abdominal tenderness, cervical motion tenderness, and foul-smelling vaginal discharge. What is the appropriate medication to start?


    A.Magnesium sulfate 4 to 6 g in 100 mL of fluid over 20 minutes, followed by 1 to 2 g per hour


    B.Ampicillin 1 g IV every 6 hours and gentamicin 15 mg per kg IV every 8 hours


    C.Ceftriaxone 250 mg IM and doxycycline 100 mg bid × 14 days


    D.Metronidazole 500 mg bid × 7 days


  Correct Answer B: Ampicillin 1 g IV every 6 hours and gentamicin 15 mg per kg IV every 8 hours. The patient has postpartum endometritis and should be admitted for broad-spectrum antibiotics. Risk factors include cesarean delivery, prolonged ruptured membranes, and younger maternal age. Magnesium sulfate would be used for treatment in severe preeclampsia or eclampsia. Rocephin and doxycycline would be appropriate outpatient treatments for pelvic inflammatory disease. Metronidazole would be an appropriate treatment for bacterial vaginosis.


132.A 32-year-old female, G4 P3003, who is 32 weeks gestation, presents with vaginal bleeding. Her vitals are as follows: temp, 98.6°F; HR, 82; BP, 120/60; RR, 18; and O2 sat 100% on RA. On exam, her abdomen is soft and nontender. What is the most appropriate next step?


    A.Discharge home and schedule a follow-up appointment with obstetrician/gynecologist in the morning


    B.Perform a sterile digital and speculum exam


    C.Order a transabdominal ultrasound


    D.Administer magnesium sulfate


  Correct Answer C: Order a transabdominal ultrasound. An ultrasound is the safest test in suspected placenta previa, which usually presents with painless bright red vaginal bleeding. Risk factors include multiparity and prior cesarean section. The placenta previa is present on ultrasound. The patient is in active labor, and arrangements should be made for immediate delivery. A speculum and digital examination should be avoided because they can lead to disruption of the placenta and can lead to bleeding. Magnesium sulfate would be used in treatment of preeclampsia.


133.A 39-year-old female, 33 weeks gestation, presents with vaginal bleeding and abdominal pain. Her medical history is significant for smoking 1 ppd. Her vitals are as follows: temp, 98.6°F; HR, 110; BP, 90/60; RR, 20; and O2 sat 100% on RA. Which of the following is false?


    A.Lab tests should include a complete blood count, type/cross match, and coagulation studies.


    B.Crystalloids should be administered.


    C.Emergency obstetrical consultation is necessary.


    D.Administer Magnesium 2 gm IV.


  Correct Answer D: Administer Magnesium 2 gm IV. The patient has abruptio placentae (or placental abruption), which is a separation of the placenta from the uterine wall. It accounts for 30% of bleeding in the second half of pregnancy. Magnesium is used in the treatment of seizures. Lab tests should be performed, crystalloids should be started, and emergent obstetric/gynecologic consultation should be obtained, because complications could include fetal death, maternal death from hemorrhage, or disseminated intravascular coagulation.


134.A 25-year-old female, 35 weeks gestation, arrives with epigastric pain and a headache. Vitals include the following: temp, 98.7°F; HR, 82; BP, 158/98; RR, 16; and O2 sat 100% on RA. She is Rh positive. Her labs were significant for proteinuria. Which of the following statements is not true?


    A.Administer Rhogam


    B.May administer magnesium sulfate IV followed by an infusion


    C.Hydralazine or labetalol may be administered.


    D.Betamethasone may be administered.


  Correct Answer A: Administer Rhogam. This patient has preeclampsia. Rhogam is administered to Rh-negative mothers. Magnesium sulfate may be used in severe preeclampsia to prevent seizures. Hydralazine or labetalol may be used for blood pressure control. Betamethasone may be given under 34 weeks of gestation to enhance fetal lung maturity.


135.A 32-year-old female presents with vaginal spotting for 3 days and left pelvic pain for 2 hours. Her last menstrual period was about 7 weeks ago. Her vitals are stable. She has left adnexal tenderness on examination. Her serum β-hCG is 2,000 mU per mL. Pelvic ultrasound shows no intrauterine pregnancy, but a left adnexal mass is present. What is the next step in management of this patient?


    A.Laparoscopy


    B.Discharge home and reexamine in the morning


    C.Methotrexate


    D.Ceftriaxone 250 mg intramuscular (IM)


  Correct Answer C: Methotrexate. This patient has an ectopic pregnancy with a triad of abdominal pain, amenorrhea, and vaginal bleeding. Methotrexate intramuscularly may be given with a treatment plan developed by the emergency department and obstetric/gynecologic department. A patient with an ectopic pregnancy should not be discharged home without treatment because of the risks of a ruptured ectopic pregnancy, hemorrhage, or death. Laparoscopy is useful in patients with suspected ectopic pregnancy and a nondiagnostic ultrasound. While laparoscopy is an option, this patient is stable and can be medically managed with methotrexate. Ceftriaxone IM is used in the treatment of Neisseria gonorrhoeae.


136.A 22-year-old female presents with vaginal itching and discharge for 2 days. Her vitals are stable, and she has no abdominal tenderness on examination. Pelvic examination reveals vaginal erythema and a frothy, malodorous discharge. What is the treatment for the patient?


    A.Fluconazole 150 mg orally as single dose


    B.Acyclovir 400 mg orally tid for 7 to 10 days


    C.Benzathine penicillin (PCN) G 2.4 million units intramuscular


    D.Metronidazole 2 g orally as a single dose


  Correct Answer D: Metronidazole 2 g orally as a single dose. This patient has Trichomonas vaginitis. It may cause vaginal irritation, pelvic pain, or a yellow-green frothy discharge. A “strawberry cervix,” punctate hemorrhages visible on the cervix, may be visible on examination. It may be diagnosed microscopically by the presence of motile flagellates. Fluconazole would treat candida vaginitis, which would reveal a “cottage cheese” discharge. Acyclovir would treat genital herpes, which would cause painful, fluid-filled vesicles which progress to shallow-based ulcers. Benzathine PCN G would treat syphilis, characterized by a painless ulcer with a clean base.


137.A 21-year-old female presents with pelvic pain and vaginal discharge for 2 days. She is sexually active with multiple partners and does not use any contraceptives. Her vitals are as follows: temp, 101.7°F; HR, 100; BP, 110/70; RR, 18; and O2 sat 100% on RA. She has adnexal tenderness, cervical motion tenderness, and mucopurulent discharge on pelvic examination. Her β-hCG is negative. What is the best treatment?


    A.Ceftriaxone 250 mg intramuscular (IM) and doxycycline 100 mg bid for 14 days


    B.Ofloxacin 400 mg bid for 14 days


    C.Metronidazole 500 mg bid for 14 days


    D.Discharge the patient home without treatment and wait for culture results


  Correct Answer A: Ceftriaxone 250 mg IM and doxycycline 100 mg bid for 14 days. This patient has pelvic inflammatory disease (PID). Neisseria gonorrhoeae and/or Chlamydia trachomatis are isolated in most cases. Ceftriaxone and doxycycline are appropriate oral/outpatient treatment regimens. Ofloxacin or metronidazole alone would not provide adequate coverage against anaerobes, gram-negative organisms, streptococci, as well as N. gonorrhoeae and C. trachomatis. PID should not be left untreated. PID is associated with serious sequelae: tubo-ovarian abscess, infertility, chronic pelvic pain, and ectopic pregnancy.


138.A 24-year-old female presents with vaginal pruritus and vaginal discharge for 3 days. Her vitals are stable. Examination reveals gray-white vaginal discharge. Clue cells are seen on a saline wet prep. Which of the following is not true about her diagnosis?


    A.It has been associated with preterm labor and premature rupture of membranes (PROM).


    B.Treatment is metronidazole 500 mg bid po for 7 days.


    C.Treatment should be delayed for pregnant patients.


    D.Fishy odor will be noted by addition of KOH to the discharge.


  Correct Answer C: Treatment should be delayed for pregnant patients. The patient has bacterial vaginosis. Treatment should not be delayed in symptomatic patients, regardless of pregnancy status, because of complications associated with preterm labor and PROM. Treatment includes metronidazole 500 mg bid orally for 7 days. In the first trimester of pregnancy, metronidazole 0.75%, one applicator intravaginally bid for 5 days may be given. The fishy odor is present when KOH is added to the discharge is known as a positive whiff (amine) test.


139.A 20-year-old female presents with pelvic pain. Exam reveals painful vesicles on an erythematous base in the vulvovaginal area. The patient has had no prior episodes. What is the best treatment for this patient?


    A.Valacyclovir 1 g bid for 7 to 10 days


    B.Metronidazole 500 mg bid for 7 days


    C.Fluconazole 150 mg po as a single dose


    D.Benzathine penicillin (PCN) G 2.4 million units intramuscular as a single dose


  Correct Answer A: Valacyclovir 1 g bid for 7 to 10 days. Patient’s presentation is consistent with herpes simplex. Primary infections present after an incubation period of 7 to 10 days, with painful vesicular lesions which progress to shallow-based ulcers. Metronidazole is used in treatment of bacterial vaginosis, which would have a frothy gray-white discharge. Fluconazole 150 mg as a single dose can be used in treatment of candida vaginitis, which would have a thick white discharge. Benzathine PCN G would be used to treat syphilis, which is characterized by a painless chancre or ulcer.


140.A 30-year-old lactating female, 4 weeks postpartum, presents with right breast pain. Her vitals are as follows: temp, 101°F; HR, 100; BP, 120/60; RR, 18; and O2 sat 100% on RA. The upper outer quadrant of her breast is warm and erythematous. What is the appropriate treatment?


    A.Vancomycin


    B.Penicillin (PCN)


    C.Dicloxacillin


    D.Metronidazole


  Correct Answer C: Dicloxacillin. The patient has postpartum mastitis, which is commonly caused by Staphylococcus aureus. Dicloxacillin would be the best choice because it is a penicillinase-resistant PCN. Vancomycin would be appropriate for methicillin-resistant staphylococcal infections. PCN would not provide appropriate coverage. Metronidazole would be helpful for infections caused by anaerobes.


141.The most commonly torn rotator cuff tendon in overuse injuries:


    A.Teres minor


    B.Teres major


    C.Infraspinatus


    D.Supraspinatus


    E.Subscapularis


  Correct Answer D: Supraspinatus. The rotator cuff is made up of four muscles—supraspinatus, infraspinatus, teres minor, and subscapularis. The supraspinatus is located on the superior aspect of the joint responsible for shoulder abduction and passes through the subacromial space. With repetitive overhead activity, this can cause stress on this muscle tendon, increasing risk of injury.


142.A 35-year-old male with a history of diabetes and a recent right shoulder sprain that has kept his shoulder in a sling for 1 week now presents with limited and painful range of motion. Exam reveals no fever or erythema, but has restricted and painful external rotation. The most likely diagnosis is:


    A.Frozen shoulder


    B.Rotator cuff tear


    C.Septic arthritis


    D.Impingement syndrome


  Correct Answer A: Frozen shoulder. Risk factors include prior injury and immobility, diabetes mellitus, connective tissue disorders. Most common finding is limited shoulder external rotation. Treatment is conservative, including nonsteroidal anti-inflammatory drugs, rest, ice, compression, elevation, but may require manipulation under general anesthesia to break up adhesions. Presentation is not consistent with septic arthritis as no fever or redness, less likely impingement syndrome, or rotator cuff injury as he has worsening pain despite immobilization.


143.A 27-year-old runner presents with atraumatic left anterior knee pain. The patient denies swelling, redness, or paresthesias. Pain worsens when going up or down stairs and recalls it being very painful recently when standing after watching a 2-hour movie. X-ray is negative for acute bony abnormality. The clinical diagnosis is most consistent with:


    A.Osteosarcoma


    B.Patellofemoral syndrome


    C.Meniscal tear


    D.Anterior cruciate ligament tear


    E.Posterior cruciate ligament tear


  Correct Answer B: Patellofemoral syndrome. This is the most common cause of anterior knee pain. “Theater sign” or pain worsening after prolonged sitting is often described. Meniscal tear typically presents with joint line tenderness. No trauma or instability makes anterior cruciate ligament or posterior cruciate ligament tear less likely. Osteosarcoma is a malignant bone neoplasm which may be seen on x-ray.


144.A 30-year-old female with history of intravenous drug use (IVDU) presents with red warm right knee and fever 101. Joint aspirate is concerning for septic arthritis. Which antibiotic is the most important to include in the initial empiric treatment?


    A.Ceftriaxone


    B.Vancomycin


    C.Azithromycin


    D.Augmentin


    E.Gentamycin


  Correct Answer B: Vancomycin. Given the patient’s history of IVDU, Methicillin-resistant Staphylococcus aureus (MRSA) must be suspected and vancomycin is the only listed option, which covers for MRSA. A second- or third-generation cephalosporin such as ceftriaxone should also be added for additional gram-negative coverage.


145.A positive Finkelstein test would be indicative of what condition?


    A.Carpal tunnel syndrome


    B.Tarsal tunnel syndrome


    C.De Quervain tenosynovitis


    D.Achilles tendon rupture


    E.Plantar fasciitis


  Correct Answer C: De Quervain tenosynovitis. This is an inflammation of the extensor pollicis brevis and abductor pollicis longus tendon sheaths thought to be due to repetitive activity. Grasping the thumb, which is quickly ulnar deviated, performs the test. Sharp pain over the distal radius would indicate a positive test.


146.Which antibiotic is linked to atraumatic Achilles tendon rupture?


    A.Cephalexin


    B.Vancomycin


    C.Doxycycline


    D.Ciprofloxacin


  Correct Answer D: Ciprofloxacin. Other fluoroquinolones are also linked and carry black-box warnings for tendonitis and tendon rupture. The exact mechanism is unclear, but may be due to interruption in joint blood supply or collagen formation.


147.What physical exam findings would be consistent with L3 to L4 lumbar disc radiculopathy?


    A.Decreased sensation lateral malleolus


    B.Weakness of great toe dorsiflexion


    C.Weakness of ankle dorsiflexion


    D.Weakness of ankle plantar flexion


  Correct Answer D: Weakness of ankle dorsiflexion. It may also find decreased patellar reflex and sensation medial malleolus. L4 to L5 would reveal weakness of great toe dorsiflexion, and L5 to S1 would be consistent with weakness of ankle plantar flexion.


148.An 8-year-old male presents with mild right hip pain and a limp for 3 weeks. There is no known trauma or inciting event. On exam, afebrile with decreased internal rotation. X-rays show a mildly flattened femoral head. What is the most likely diagnosis?


    A.Transient synovitis


    B.Septic arthritis


    C.Legg–Calvé–Perthes syndrome


    D.Slipped capital femoral epiphysis


  Correct Answer C: Legg–Calvé–Perthes syndrome. The pain and limp are due to avascular necrosis of the femoral head, sometimes not seen on x-ray and may require MRI for further evaluation. Although slipped capital femoral epiphysis can present similarly, x-rays show dissociation of the femoral epiphysis with the metaphysis that may result in avascular necrosis, and most cases require orthopedic surgical treatment. This presentation is not consistent with transient synovitis or septic arthritis as no history of fever or recent viral illness.


149.A patient presents to the emergency department complaining of hip pain. When asked further questions regarding this pain, the patient trails off telling unrelated stories about irrelevant topics, but eventually returns to the original subject at hand. What process is this person demonstrating?


    A.Circumstantiality


    B.Tangentially


    C.Flight of ideas


    D.Mania


  Correct answer A: Circumstantiality. This implies that the original topic or question is returned to despite deviating from this topic along the way. Tangentially is incorrect because it means the person will go off on a tangent of topics, but does not return to the original topic of discussion. Flight of ideas implies that the person will discuss whatever topic enters his or her brain spontaneously, even if they are unrelated to each other. Mania is characterized by a state or period of intense elation, and often lack of sleep, grandiose gestures, and feeling invincible.


150.A patient presents to the emergency department complaining of foot pain. He is found to have multiple recent visits for various different complaints that have been worked up with negative results. The patient is convinced he has a medical disorder that is causing these unrelated issues and continued to have new complaints frequently. What is the most likely diagnosis?


    A.Obsessive compulsive disorder


    B.Hypochondriasis


    C.Panic disorder


    D.Schizophrenia


  Correct answer B: Hypochondriasis. This is a condition in which individuals are convinced they have a variety of health concerns when there is often no diagnosis found. Obsessive compulsive disorder is often characterized by impulsive gestures that are irrational and the person recognizes as such. Panic disorder is when patients suffer from frequent panic attacks that can be triggered by a variety of scenarios. Disorganized thinking, delusions, and hallucinations characterize schizophrenia.


151.Which of the following medications can precipitate a manic event in an individual with suspected depression?


    A.Olanzapine


    B.Aripiprazole


    C.Sertraline


    D.Nortriptyline


  Correct answer C: Sertraline. Patients with an underlying history (or family history) of bipolar disorder can be thrown into a manic event when given selective serotonin reuptake inhibitors (SSRIs) for treatment of depression. The mechanism by which this happens is directly related to the inhibition of serotonin reuptake in the synaptic space. Patients who have had a suspected manic episode in the past should not be started on SSRIs as initial management of depression. The remaining medications in this list act on different neurotransmitters (or less heavily on serotonin) and are therefore unlikely to propagate a manic episode.


152.A 38-year-old female is brought to the emergency department after she was missing for several days. She did not return back to her home after dropping her kids off from school one morning. She was found in the next town over from her home. When questioned about her disappearance, the woman provides you with a different name and profession, and is unable to identify her family members. What is the most likely diagnosis?


    A.Bipolar disorder


    B.Dissociative fugue


    C.Dementia


    D.Histrionic personality disorder


  Correct answer B: Dissociative fugue. Dissociative fugue is when an individual removes himself/herself from his or her prior identity and lifestyle, and assumes a new identity. Bipolar disorder involves both episodes of depression and mania, but does not mean people change their names, profession, etc. Dementia is often age related and occurs gradually over time. Histrionic personality disorder is characterized by excessive attention-seeking behavior and attempted manipulation.


153.Which of the following trio of symptoms suggests a diagnosis of schizophrenia?


    A.Depression, anxiety, suicidal ideation


    B.Flight of ideas, word salad, tics


    C.Disorganized thinking, grandiose gestures, emotionally labile


    D.Auditory hallucinations, visual hallucinations, disorganized thinking


  Correct answer D: Auditory hallucinations, visual hallucinations, disorganized thinking. Hallucinations (both auditory and visual) and disorganized thinking or behavior are characteristic traits of schizophrenia. Depression, anxiety, and suicidal ideation can be linked to major depressive or generalized anxiety disorder. Option B includes speech characteristics that collectively do not pertain to a specific diagnosis. Individually, tics often occur in individuals with Tourette syndrome. Grandiose gestures are often used to describe someone’s activity when he or she is in a manic episode. Emotional lability describes incontrollable crying or laughing that often occurs after a brain injury.


154.A mother brings her 8-year-old daughter to the emergency department with the complaint that she has been having diarrhea. The child appears dehydrated and malnourished; so she is admitted to the hospital for further workup. Upon further investigation, the child has been brought to her pediatrician multiple times for the same complaint, and has repeatedly given a diagnosis of acute gastroenteritis. While admitted to the hospital, the child has a single episode of nonbloody, watery diarrhea that quickly resolves after admission. On hospital day 2, she is able to tolerate food and liquids without any difficulty. Her hospital workup shows normal blood work and negative stool cultures. Immediately after returning home, the mother calls the pediatrician’s office again complaining that the child’s diarrhea has returned, is now associated with nonbilious nonbloody vomiting, and the hospital must not have done their job correctly. What is the most likely diagnosis?


    A.Ulcerative colitis


    B.Crohn disease


    C.Munchausen by proxy


    D.Foodborne illness


  Correct answer C: Munchausen by proxy syndrome. This syndrome occurs when individuals create or fabricate health concerns for individuals they take care of. In this scenario, the mother is thought to be inducing diarrhea in her daughter, since her symptoms completely resolve once the child is under someone else’s care. In addition, her negative workup suggests that this diarrhea would resolve on its own if it was not self-inflicted. Ulcerative colitis and Crohn’s disease are often characterized by bloody diarrhea and are accompanied by abdominal pain, bloating, and elevated white blood cell count. Foodborne illness often occurs approximately 6 hours after ingesting the affected substance, and usually resolves after 24 hours. In addition, foodborne illness might have positive stool culture results in the case of Salmonella, Campylobacter, Shigella, etc.


155.Which of the following medications should be avoided in an individual with a known prolonged QT interval?


    A.Ondansetron


    B.Quetiapine


    C.Levofloxacin


    D.All of the above


  Correct answer D: All of the above. All of the listed medications can potentially further elongate someone’s QT interval, resulting in sudden arrhythmia and sometimes death.


156.A woman goes to her doctor’s office and demands to be seen immediately by a practitioner, claiming she is the Queen of England and has a serious health issue. She is told that she must call to make an appointment due to the office’s booked schedule, but she refuses to leave. She begins to cause a disturbance in the waiting room; so security is called to escort her out of the office. She continues to discuss her royal stature and attempts to flirt with the security guard in order to divert the trouble. After being escorted out of the office, the patient gets extremely agitated and begins weeping in the lobby of the building. What is the most likely diagnosis?


    A.Borderline personality disorder


    B.Histrionic personality disorder


    C.Conversion personality disorder


    D.Bipolar personality disorder


  Correct answer B: Histrionic personality disorder. This is characterized by excessive attention-seeking behavior and attempted manipulation, as this patient exhibits by claiming to be the Queen of England. It is often accompanied by exaggerated and frequently changing emotions, as exemplified by her flirtation that quickly turned to agitation. Borderline personality disorder is often described by one’s interpersonal relationships. Individuals with this diagnosis have difficulty maintaining friendships, have fear of abandonment, and have difficulty handling their emotions. Conversion personality disorder often presents as a loss in sensory or motor function that is unexplained by a medical workup. Bipolar disorder is characterized by episodes of both depression and mania.


157.What is the recommended first-line therapy in someone with newly diagnosed generalized anxiety disorder (GAD)?


    A.Atypical antipsychotics


    B.Benzodiazepines


    C.Tricyclic antidepressants


    D.Low-dose selective serotonin reuptake inhibitors (SSRIs)


  Correct answer D: Low-dose selective serotonin reuptake inhibitors (SSRIs). These have been found to be effective in treating GAD, and are recommended as the initial therapy. Benzodiazepines are often used in conjunction with SSRIs for breakthrough anxiety, but alone are not sufficient for treatment of GAD. In addition, benzodiazepines have several side effects that prevent them from being used on a regular basis (such as sedation, psychomotor impairment, and withdrawal symptoms). Tricyclic antidepressants are often used to treat major depressive disorder in patients who have failed SSRIs. Atypical antipsychotics are traditionally used to treat personality disorders and are not indicated as treatment for GAD.


158.A 55-year-old male is brought into the emergency department by emergency medical services after being found unresponsive on a park bench. Upon arrival to the emergency department, the patient appears disheveled, unkempt, and smells of alcohol. The patient responds to aggressive sternal rub, but otherwise appears somnolent. His blood alcohol level returns at 405. The patient has not been treated at this hospital before, and prior medical records are unable to be obtained. After several hours in the emergency room, one of the nurses comes out of his room and reports he appeared tremulous, diaphoretic, and agitated. What is the best next step in the care of this patient?


    A.Administer intravenous lorazepam immediately


    B.Administer intranasal Narcan


    C.Give a 1 L bolus of IV fluid


    D.Discharge the patient home


  Correct answer A: Administer intravenous lorazepam immediately. The patient is exhibiting signs of alcohol withdrawal. The appropriate management for alcohol withdrawal in an acute setting is benzodiazepines, as they decrease the symptoms of withdrawal by enhancing γ-aminobutyric acid receptor activity. If the patient continued to withdraw from alcohol and no treatment was administered, he would be at risk for experiencing withdrawal seizures. Narcan is useful when it is suspected that someone has overdosed on opioids, and is ineffective at treating symptoms of alcohol withdrawal. Administering IV fluids in a patient who is intoxicated is helpful to achieve hydration, but will not have any effect on symptoms of alcohol withdrawal. Discharging the patient home is an inappropriate option given the patient could experience delirium tremens if not properly treated.


159.A 25-year-old male presents to the emergency department with “a black eye.” He states last night he was in a fight with his roommate and was punched in the left eye. Today he woke up with blurry vision and pain. On exam you note ecchymosis and edema of the periorbital tissues, subconjunctival hemorrhage, and that the left eye remains fixed when the patient gazes upward. What condition are you concerned about?


    A.Entrapment of the inferior oblique muscle


    B.Blowout fracture with entrapment of the inferior rectus muscle


    C.Mandible fracture with inferior rectus and lateral rectus muscle rupture


    D.Blowout fracture with entrapment of the superior rectus muscle


  Correct Answer B: Blowout fracture with entrapment of the inferior rectus muscle. This question tests the examinee’s ability to correctly identify the disorder and its complications. Options A and D are incorrect as these muscles do not control upward gaze. Option C is incorrect as there is no indication in the history of a mandibular fracture.


160.The patient from question 1 has a CT scan showing fracture of the medial wall and orbital floor with air. What is the appropriate treatment course and discharge plan?


    A.Transport to the operating room for emergent surgical intervention


    B.Discharge home with referral to ENT


    C.Discharge home with sinus precautions, decongestants, Augmentin 875/125 bid × 7 days, and plan for surgery within 2 weeks


    D.Discharge home with nasal splint, oxycodone, and erythromycin ointment


  Correct Answer C: Discharge home with sinus precautions, decongestants, Augmentin 875/125 bid × 7 days, and plan for surgery within 2 weeks. This question relies on the examinee’s ability to correctly identify the injury and its treatment plan. Isolated blowout fractures are rarely operative and certainly not urgently. We do not splint these fractures, and although ENT referral is indicated, there are other instructions the patient needs.


161.A 35-year-old male is brought to the emergency department from a softball game with coworkers holding his face. He tells you that he walked behind a friend swinging a bat and was struck in the face with a metal bat. He “blacked out for a few seconds.” He is holding a bloodstained shirt to his face and is complaining of severe facial pain, and has no other complaints. You obtain a maxillofacial CT scan and note a pyramidal fracture of the central maxilla and palate. What type of fracture is this?


    A.Nasal fracture


    B.LeFort I


    C.LeFort II


    D.LeFort III


    E.LeFort IV


  Correct Answer C: LeFort II. LeFort II fracture is pyramidal fracture of the central maxilla and palate. A nasal fracture involves only the nasal bones. LeFort I is a transverse fracture that separates the maxilla from the lower portion of the pterygoid plate and nasal septum. LeFort III or craniofacial dissociation is a fracture line through the nasofrontal and frontomaxillary sutures, extending along the medial wall of the orbit through the ethmoid bones, along the floor of the orbit to the zygomatic arch. Facial tugging causes the entire face. LeFort IV fractures include the frontal bone and midface.


162.A 75-year-old male with a medical history of atrial fibrillation, hypertension, and non-insulin-dependent diabetes is brought in by ambulance after an unwitnessed fall. He states he was walking in his kitchen when he “fell out.” The next thing he remembers is his wife standing over him shouting. Emergency medical services reports that his vital signs in the field are as follows: temp, 97.4°F; HR, 130; irregular BP, 100/60; RR, 20; and O2 sat 95% on RA. Upon walking into the room you find the patient on a backboard with C-spine immobilization, awake, moaning in obvious distress with the right leg externally rotated and shortened. Your next step is:


    A.Obtain IV access and ECG and give fluid bolus


    B.Obtain vital signs, check pulses in all limbs, and complete the secondary survey


    C.Give 10 mg of oxycodone and Tylenol


    D.Place the right leg in traction


  Correct Answer B: Obtain vital signs, check pulses in all limbs, and complete the secondary survey. This is the next step in the advanced trauma life support algorithm. The vital signs listed were pretransport. In addition, the examinee should recognize this patient has a potential hip fracture and/or dislocation, which could compromise the affected limb. Checking pulses is the first step in identifying vascular compromise. Getting IV access, EKG, putting leg in traction are all necessary actions, but not the next step in the algorithm. This patient should be kept NPO until further evaluation; so he should not receive oxycodone and Tylenol.


163.The family of the above patient is concerned that the C-spine collar is making the patient too uncomfortable and wants you to remove it. Do you need to image this patient’s C-spine?


    A.Yes, because he has an altered level of consciousness


    B.Yes, because he has a distracting painful injury


    C.No, he is awake and alert and does not have any midline C-spine tenderness


    D.Both A and B


  Correct Answer B: Yes, because he has a distracting painful injury. This question tests the examinee’s knowledge of Nexus criteria for C-spine imaging, which states the C-spine of a trauma can be safely cleared without imaging if none of the following are present: Focal neurologic deficit, midline spine tenderness, altered level of consciousness, intoxication, distracting injury.


164.To provide optimal analgesia for the above patient and continue to adequately care for the patient, you:


    A.Give 100 µg fentanyl and 2 mg midazolam


    B.Subcutaneous morphine 2 mg every 1 hour up to three doses


    C.Ultrasound-guided femoral nerve block with 0.5% bupivacaine


    D.Intubate using rapid sequence intubation and sedate with propofol


  Correct Answer C: Ultrasound-guided femoral nerve block with 0.5% bupivacaine. The block allows you to use safe, effective, and resource-conscious analgesia. It is minimally invasive, and provides excellent sustained analgesia without sedation or airway compromise. Fentanyl and midazolam serve as both an analgesic and sedative medication effectively inducing conscious sedation. This would require extra nursing staff to monitor the patient and puts the patient at risk for airway compromise. Although sedation may be necessary to reduce this patient’s hip, adequate analgesia can be obtained prior to the reduction procedure without sedating the patient. Morphine 2 mg subcutaneous is incorrect as it is too low a dose to provide analgesia for this patient’s injuries. This patient is protecting his airway with a normal mental status, and intubation should not be the first management strategy for this patient.


165.An 85-year-old female comes to the emergency department with chest wall pain after a fall forward hitting her right chest on the corner of her nightstand 3 days ago. She has taken Tylenol without any improvement. Her chest x-ray shows nondisplaced fractures of the right ribs 3, 4, and 5 and small right lower lobe patchy opacity concerning for atelectasis or early pneumonia. Your plan is:


    A.Place a Lidoderm patch to the area, advise the patient to take Tylenol 1,000 mg every 8 hours alternating with ibuprofen 800 mg every 8 hours, prescribe a azithromycin 5-day pack, and schedule follow-up in 1 week with her primary care physician (PCP).


    B.Prescribe oxycodone/acetaminophen 5/500 1 to 2 tabs every 6 hours, ibuprofen 800 mg every 8 hours, and azithromycin 5-day pack and schedule follow-up in 3 days with her PCP.


    C.Admit to the hospital for pain management and antibiotics and respiratory physical therapy.


    D.Either A or B


  Correct Answer C: Admit to the hospital for pain management and antibiotics and respiratory physical therapy. This patient is at high risk for complications of her rib fractures and may already have a small pneumonia. As she is elderly, she is at risk for oversedation with opiate analgesics and should be admitted to the hospital for close monitoring, pain control, and antibiotics.


166.On your way home from work, you arrive at the scene of a high-speed motor vehicle crash. Paramedics are already on scene, and you offer your assistance. Upon evaluation of the unbelted driver of the vehicle, you find him lying on the ground, anxious, gasping for breath, a respiratory rate of 35, with tracheal deviation to the right and distended neck veins. You correctly identify the patient has a


    A.Tension pneumothorax


    B.Pulmonary contusion


    C.Fractured C-spine


    D.Cardiac contusion


  Correct Answer A: Tension pneumothorax. This patient has clinical signs of a tension pneumothorax.


167.On the above patient, you immediately perform the following intervention improving his clinical status.


    A.Needle decompression of the right lung with 14-gauge IV catheter in the second intercostal space midclavicular line


    B.Apply C-spine immobilization


    C.Needle decompression of the left lung with 14-gauge IV catheter in the second intercostal space midclavicular line


    D.Tube thoracostomy


  Correct Answer C: Needle decompression of the left lung with 14-gauge IV catheter in the second intercostal space midclavicular line. In tension pneumothorax, tracheal deviation points away from the lung under tension (the left in this case) and that needle decompression is the first choice when a tube thoracostomy cannot be placed immediately (as is the case in this scenario). C-spine immobilization may be necessary for this patient, but does not treat the immediate life-threat of tension pneumothorax. A tube thoracostomy cannot be accomplished in a safe, clean, and fast manner on this patient.


168.A 25-year-old female is brought in from the scene of a motor vehicle crash with chest pain. Paramedics tell you the patient was the belted driver of a sedan when she fell asleep and struck a telephone pole at 45 mph. Airbags did deploy. The patient has been having escalating chest pain during transport with minimal improvement after morphine 2 mg IV. Vital signs are as follows: temp, 97.8°F; HR, 140; BP, 90/50; RR, 22; and O2 sat 95% on RA. She is awake, but pale. You order a screening portable chest x-ray that shows an abnormal mediastinum and a hemothorax increasing your suspicion for


    A.Aortic dissection


    B.Cardiac contusion


    C.Rib fracture


    D.None of the above


  Correct Answer A: Aortic dissection. The most life-threatening injury that could result from rapid deceleration described in this scenario and the clinical symptoms and signs are of aortic dissection/rupture. A motor vehicle accident with chest wall trauma could cause cardiac contusion and rib fracture, but the chest x-ray findings of an “abnormal mediastinum” should clue the examinee to think of aortic dissection.


169.To confirm the injury in the questions above, you order which of the following:


    A.Thoracotomy


    B.CT-angiogram of the chest/abdomen/pelvis


    C.MRI of the chest/abdomen/pelvis


    D.Transesophageal echocardiogram


  Correct Answer B: CT-angiogram of the chest/abdomen/pelvis. This test will give the most rapid and extensive evaluation of this patient’s thorax. Thoracotomy is incorrect as it is too invasive for this semistable patient. MRI and echo require additional resources and technicians to perform and are therefore too time consuming for this patient scenario.


170.Emergency medical services arrives with a 40-year-old otherwise healthy male who fell off a 10-ft ladder landing on his feet. He has a Glasgow Coma Scale of 15 and stable vital signs. You confirm that he has bilateral calcaneal fractures. What other injury is most often associated with his injuries?


    A.Rib fracture


    B.Lumbar spine fracture


    C.Concussion


    D.Bladder rupture


  Correct Answer B: Lumbar spine fracture. This is a dangerous mechanism and often has associated spinal fractures. It is important to think of other areas where the patient might have injuries when he has another painful injury. Although a rib fracture and concussion may be present, these are not the most common associated injuries. Bladder rupture is incorrect because the patient would be in more distress and have other notable physical exam findings.


171.A 32-year-old male patient presents after falling off his snowmobile at 15 mph. He was helmeted and did not lose consciousness. He is awake and alert with severe pain in his left leg and an open distal tibia-fibular fracture. Vital signs are as follows: temp, 97.8°F; HR, 120; BP, 100/70; RR, 24; and O2 sat 96% on 2 L by nasal cannula. You perform a bedside _______, which identifies fluid between the liver and kidney and the spleen and kidney.


    A.Focused assessment with sonography for trauma (FAST)


    B.Plain film kidney, ureter, and bladder and upright chest


    C.CT-angiogram of the chest/abdomen/pelvis


    D.Diagnostic peritoneal lavage


  Correct Answer A: Focused assessment with sonography for trauma (FAST). A FAST is a bedside procedure that is standard in the current evaluation of the trauma patient. Relevant findings of these tests are the only routine bedside procedure that specifically identifies fluid in these two locations.


172.While you are doing the focused assessment with sonography for trauma, the blood pressure drops to 90/65. Your next step is:


    A.Call the general surgeon and arrange for urgent exploratory laparotomy


    B.Call the orthopedic surgeon and arrange for urgent repair of his open fracture


    C.Give a bolus of normal saline IV fluids


    D.Given fentanyl 100 µg and reduce the ankle


  Correct Answer C: Give a bolus of normal saline IV fluids. Giving IV fluids is the correct next step based on advanced trauma life support guidelines. You should be concerned that the patient is clinically deteriorating, likely has an intra-abdominal hemorrhage, and needs urgent intervention. Calling the surgeon for an ex-lap will most likely be necessary, but the clinician must first treat the patient, then coordinate disposition. Calling ortho and giving fentanyl are incorrect as the ankle fracture is not the priority injury at this time is unlikely the cause of the patient’s hypotension.


173.You are caring for an elderly man with a recent fall from a small (3 step) stepladder. He is complaining of deep hip pain and an inability to walk. The nurses call your attention to bleeding at the urethral meatus. This finding confirms your suspicion for what two related injuries?


    A.Dementia and bladder infection


    B.Intra-abdominal bleeding and femur fracture


    C.Bladder rupture and hip fracture


    D.Urethral injury/tear and pelvic fracture


  Correct Answer D: Urethral injury/tear and pelvic fracture. The patient has clinical signs of a urethral injury. Dementia and bladder infection are more likely to present in a more insidious manner and not the most acutely concerning illness. Options B and C include high-risk injuries that occur together with significant trauma, which is not the case with this scenario.


174.A 15-year-old boy is brought to your community emergency department (ED) after a particularly hard hit on the football field. He tells you he passed out after he was tackled and bystanders noted he was unresponsive for about 15 seconds. In the ED he is awake and alert, but cannot tell you his name, the date, or his home address; he has equal grip strength in both the upper and lower extremities and can hold both arms and legs off the stretcher for 4 seconds. This patient has a Glasgow Coma Scale of:


    A.14


    B.16


    C.13


    D.10


  Correct Answer A: 14. This question tests the ability to apply the standard scale used to indicate a patient’s level of consciousness. The patient gets a 4 for eye response, 4 for verbal response, and 6 for motor response for a total of 14.


175.On the above patient, you decide to do a CT scan of the brain, which has no evidence of hemorrhage or skull fracture. You do which of the following?


    A.Transfer to a trauma center for neurology evaluation


    B.Discharge home with his parents and do urgent follow-up in the pediatric concussion clinic with clear return instructions and no sports until reevaluation


    C.Observe for 6 hours and repeat the head CT


    D.None of the above


  Correct Answer B: Discharge home with his parents and do urgent follow-up in the pediatric concussion clinic with clear return instructions and no sports until reevaluation. As this patient does not have any evidence of intracranial hemorrhage, he does not need urgent neurosurgical or neurological intervention, although he does need close monitoring by a family or friend and clear return precautions. The examinee should also recognize the importance of subsequent head trauma and restrict this patient from high-risk sports, such as football.


176.You are examining one of the homeless men who frequently comes to your community emergency department intoxicated with alcohol. Typically, he is pleasant and cooperative despite his intoxication. Today, he is combative, has no odor of alcohol, and his gate is more unsteady than baseline. You are concerned and order a CT scan of the brain. What finding warrants acute neurosurgical evaluation?


    A.Large chronic subdural hematoma with acute bleeding and 3 mm of midline shift


    B.Large chronic subdural hematoma


    C.White matter changes and lacunar infarcts of undetermined age


    D.Large territorial ischemic stroke


  Correct Answer A: Large chronic subdural hematoma with acute bleeding and 3 mm of midline shift. This is the only choice that necessitates urgent neurosurgical evaluation and evacuation of the hemorrhage. Large territorial ischemic stroke necessitates an urgent neurological evaluation.


177.Emergency medical services brings in a 50-year-old male with complaints of neck pain and paresthesias of both arms after a 30-lb barrel fell onto his head from a shelf. He has no other medical problems and takes no medications. His initial exam is notable for vital signs of the following: temp 98.7°F, HR 100, BP 130/80, RR 20, O2 sat of 97% on RA, a Glasgow Coma Scale of 14, moderate confusion, tenderness of C3-4, and diminished sensation on bilateral upper extremities. What initial imaging studies do you obtain?


    A.Chest x-ray and pelvis x-ray


    B.MRI/MRA brain


    C.CT brain and CT C-spine


    D.All of the above


  Correct Answer C: CT brain and CT C-spine. This patient has neurological injuries after an axial load to the skull that may have caused either intracerebral trauma or cervical spine injury. Option A does not address this patient’s neurologic complaints, and option B does not include cervical imaging.


178.Imaging on the above patient reveals a Jefferson fracture (C1 burst fracture). Your next step is:


    A.Clinically clear the patient from C-spine immobilization and discharge with orthopedics follow-up in 3 days


    B.Consult a spine surgeon


    C.Give oral pain medicines, oxycodone, ibuprofen, and Tylenol until the patient’s pain improves


    D.Obtain imaging of the remainder of the patient’s spine


    E.Both B and D


  Correct Answer E: Both B and D. This patient has potentially unstable spine fractures, and there may be additional injuries not noted on the first exam. A C1 fracture is potentially unstable and needs urgent evaluation by a spine surgeon so that immobilization should not be cleared. Oral pain medications are incorrect, as this patient should remain NPO until evaluation by a surgeon for operative planning.


179.A 32-year-old female, with a history of polycystic ovary syndrome, hypertension, hyperlipidemia, and a body mass index >40, presents to the emergency department with a severe headache and requires a lumbar puncture for concern of pseudotumor cerebri. What is the best method for performing a lumbar puncture on this patient?


    A.By assuming the fetal position with the spine parallel to the bed


    B.In the seated position facing away from the operator with spine perpendicular to the bed


    C.In the seated position bending over a bedside table to maximize lumbar flexion


    D.Place in the decubitus position after the spinal needle has been placed in the seated position over a bedside table tray


  Correct Answer D: Place in the decubitus position after the spinal needle has been placed in the seated position over a bedside table tray. Positioning is a key aspect of performing a lumbar puncture. In obese patients, the best way to achieve maximal lumbar flexion is in the seated position bending over a bedside table. However, cerebrospinal fluid pressure, which is diagnostic of pseudotumor cerebri, is only properly assessed in the supine position. Therefore, once the needle has been inserted into the spinal canal, the patient can be carefully repositioned lie in the decubitus position to accurately measure pressure.


180.Roughly 5% to 40% of patients develop a postdural headache after a lumbar puncture (LP) is performed. What is the best way to prevent postdural puncture headache?


    A.Using the smallest gauge needle


    B.Place patient in seated position while performing LP


    C.Immediately lay patient supine after procedure for 1 hour


    D.Prophylactically fluid bolus the patient before the procedure


  Correct Answer A: Using the smallest gauge needle. The incidence of postdural headache is directly correlated with needle gauge, thus using the smallest gauge needle can help minimize this occurrence. The patient’s position during and immediately following LP does not appear to be associated with the development of prevention of a headache, nor does administration of a periprocedure fluid bolus.


181.Which of the following is not the standard of care in verifying the placement of a nasogastric tube (NG)?


    A.Chest x-ray


    B.Insufflation of air, resulting in borborygmi over the epigastrium


    C.Aspiration of contents


    D.Normal clear speech


  Correct Answer A: Chest x-ray. Radiographic verification is the most sensitive test to detect proper placement of NG tube, but not the standard of care. The best methods of verifying placement are air bubbles heard within the stomach, aspiration of gastric contents, and in a conscious patient a normal speech without coughing.


182.A 35-year-old male is brought in by ambulance after a high-speed motor vehicle crash with direct head trauma against steering wheel. The patient is alert and oriented, with multiple midface fractures seen on CT. There is concern for a transdiaphragmatic hernia, and placement of a nasogastric tube (NGT) is warranted. Supplies are gathered at the bedside. What is the next best step in placement of this NGT?


    A.Estimation of tube length by measuring from patient’s tip of the nose to earlobe, to the xiphoid process


    B.Assessing for patency of nares


    C.Anesthetizing the oropharynx with lidocaine 4%


    D.None of the above


  Correct Answer D: None of the above. Placement of NGT in patients with cribriform plate or midface fractures is a contraindication given the susceptibility to intracranial placement of NGT. Therefore, the NGT should not be placed in this patient and alternative methods should be discussed. Typically, estimation of tube length is the first step in placement of an NGT.


183.During the insertion of a chest tube after the patient has been properly positioned, what are the appropriate steps?


    A.Sterilize the skin over insertion site on the anterior thorax and midaxillary line, drape the surrounding area, anesthetize, and make an incision over the superior aspect of the rib


    B.Sterilize the skin over insertion site on the lateral thorax and anterior axillary line, drape the surrounding area, anesthetize, and make an incision over the inferior aspect of the rib


    C.Sterilize the skin over insertion site on the anterior thorax and posterior axillary line, drape the surrounding area, anesthetize, and make an incision over the inferior aspect of the rib


    D.Sterilize the skin over insertion site on the lateral thorax and midaxillary line, drape the surrounding area, anesthetize, and make an incision over the superior aspect of the rib


  Correct Answer B: Sterilize the skin over insertion site on the lateral thorax and anterior axillary line, drape the surrounding area, anesthetize, and make an incision over the inferior aspect of the rib. Chest tubes are inserted in the lateral thorax at the anterior axillary line, just lateral to the nipple at the level of 4th or 5th intercostal space, ensuring that it is above the diaphragm. Incisions must be made on the inferior aspect of the rib to avoid the neurovascular bundle that is located superior to the rib.


184.A 60-year-old male with a history of benign prostatic hyperplasia (BPH) was brought into the emergency department after mechanical slip and fall with obvious left hip deformity and an unstable pelvis. A Foley catheter is needed. What should you do in preparation for placing this Foley catheter?


    A.Grab a 14F coudé catheter


    B.Lubricate the urethra


    C.Perform a prostate exam


    D.Sterilize the glans and urethral meatus


  Correct Answer C: Perform a prostate exam. A digital rectal exam must be performed before a catheter is inserted in a male patient with major blunt trauma. Specifically feel for free-floating prostate or gross blood escaping from the urethra, which signifies urethral rupture and is an absolute contraindication to Foley placement. Generally, patients with BPH require 20 to 22F coudé catheter. Lubricating and sterilizing the urethra are necessary steps in Foley catheter placement, which are typically performed after donning sterile gloves.


185.Which statement is false regarding arterial blood gas (ABG) sampling?


    A.Allen test must be performed before sampling.


    B.Can be performed in the same arm as arteriovenous (AV) fistula.


    C.Needle is held at a 45-degree angle aiming caudally, with wrist in extension.


    D.Air bubbles must be removed from the syringe for accurate analysis.


  Correct Answer B: Can be performed in the same arm as arteriovenous (AV) fistula. The presence of AV fistulas and vascular grafts are a contraindication to ABG sampling of that same arm. Allen test must be performed to assess adequate collateral blood flow of the hand. Air bubbles that are left in the syringe can alter PO2 and PCO2 values and lead to false interpretation; therefore, these must be removed prior to analysis.


186.What are the appropriate steps in performing a central venous catheter?


    A.Use ultrasound to assess patency of internal jugular vein, insert needle into vein, place guide wire through lumen of the needle, using scalpel make small incision in the skin where the wire penetrates, advance dilator over the wire and into the lumen of the vein, advance the catheter over the guide wire, remove the guide wire


    B.Insert needle into vein, use ultrasound to assess patency of internal jugular vein, using scalpel make small incision in the skin where needle penetrates, advance guide wire through the lumen of the needle, use dilator over wire and into the lumen of the vein, advance the catheter over the guide wire, remove the guide wire


    C.Use ultrasound to assess patency of the vein, insert needle in internal jugular vein, place guide wire through the lumen of the needle, advance catheter over guide wire, using scalpel make small incision in the skin where the catheter penetrates, remove the guide wire


    D.Insert needle into vein, use ultrasound to assess patency of internal jugular vein, place guide wire through lumen of the needle, using scalpel make small incision in the skin where the wire penetrates, advance dilator over the wire and into the lumen of the vein, advance the catheter over the guide wire, remove the guide wire


  Correct Answer A: Use ultrasound to assess patency of internal jugular vein, insert needle into vein, place guide wire through lumen of the needle, using scalpel make small incision in the skin where the wire penetrates, advance dilator over the wire and into the lumen of the vein, advance the catheter over the guide wire, remove the guide wire. Ultrasound must be performed first to assess patency of the vein as thrombus or stenosis of the vessel is contraindication to performing central venous access. Needle is inserted; guide wire is placed through the lumen of the needle, and needle is removed. A scalpel is used to make incision; a dilator is passed over the guide wire to create a tract for the pliable catheter to follow. The dilator is removed, the catheter is advanced over the wire and into the lumen of the vein, and the guide wire is removed. This method is referred to as the Seldinger technique.


187.Which of the following is true regarding wound care?


    A.Closed wounds are best treated with moist dressings


    B.Open wounds are best treated with dry dressings


    C.Petrolatum dressings (Xeroform) decrease the rate of epithelialization in partial-thickness wounds


    D.Immobilization of the wound may delay wound healing


  Correct Answer C: Petrolatum dressings (Xeroform) decrease the rate of epithelialization in partial-thickness wounds. Partial-thickness wounds are better treated with ointment dressings (Bacitracin, Neosporin) or nonadherent occlusive dressings. Closed wounds are best treated with dry dressings, because moist dressings cause maceration of the skin and invite bacterial invasion. Open wounds are best treated with moist dressings to the open wound surface and dry dressings on top to achieve a capillary effect. Immobilization of the wound enhances resistance to infection and actually accelerates wound healing.


188.A 22-year-old male, otherwise healthy, presents to the emergency department after sustaining a laceration to his thumb using electric hedge clippers. What is the preferred method of wound irrigation?


    A.Hydrogen peroxide


    B.Normal saline


    C.Ionic soap/detergent


    D.1% povidone-iodine


  Correct Answer B: Normal saline. Washing the open wound with saline under pressure removes most of the surface bacteria. Hydrogen peroxide may be useful for removing debris and blood clots; however, there is a risk of cytotoxicity. Ionic soap and detergents are extremely irritating to tissues and increase the potential for infection if used directly on the wound. The use of concentrated iodine has been associated with a slightly decreased rate of wound infection but at risk of cytotoxicity. When diluted, there is no advantage to using this over normal saline.


189.Which of the following is contraindication to rapid sequence intubation (RSI)?


    A.Cardiac arrest


    B.Oral trauma


    C.Angioedema


    D.Recent mandibular open reduction internal fixation


    E.All of the above


  Correct Answer E: All of the above. With cardiac arrest, there is no indication for muscle relaxation/paralysis. With oral trauma and mandibular fixation, there is a high likelihood of difficulty with bag valve mask ventilation if the RSI fails and there is no backup rescue ventilation option. With angioedema, first attempt should be made with an awake nasal intubation with plan for surgical airway as the backup.


190.A 90-year-old female presents to the emergency department (ED) with onset of palpitations, shortness of breath, and chest pressure that started 30 minutes prior. Emergency medical services found her with a heart rate of 160, BP of 110/74, respiratory rate of 26, and O2 saturation of 96% on room air. On arrival to the ED she is diaphoretic, pulse is 170, and BP is 70/palp. EKG reveals an irregularly irregular narrow-complex tachycardia. The next treatment would be:


    A.Diltiazem 20 mg intravenous pyelogram (IVP)


    B.Metoprolol 5 mg IVP


    C.Adenosine 6 mg IVP


    D.Synchronized cardioversion at 100 J


    E.Unsynchronized cardioversion at 200 J


  Correct Answer D: Synchronized cardioversion at 100 J. In a hemodynamically unstable patient, the treatment of choice is synchronized cardioversion. Unsynchronized cardioversion (option E) for atrial fibrillation can cause other arrhythmias. Medications (options A, B, and C) are not first-line treatment for new onset of hemodynamically unstable atrial fibrillation.


191.Which of the following patient is most appropriate for transcutaneous pacing?


    A.Asystolic cardiac arrest


    B.Hypothermic drowning victim with pulse 30 and BP 80/60


    C.Patient with second-degree Mobitz atrioventricular (AV) block and BP 70/40


    D.Patient with third-degree AV block and BP 120/80


  Correct Answer C: Patient with second-degree Mobitz atrioventricular (AV) block and BP 70/40. The most appropriate patient is the patient with a high-degree AV block and unstable vital signs or clinical course. There has been no data to support benefit from pacing an asystolic cardiac arrest (option A). In hypothermia (option B), the heart is unable to respond to the electrical stimulus. In option D, the patient is hemodynamically stable and transcutaneous pacing is not the best first treatment choice.


192.Which of the following is not associated with a chest x-ray finding indicative of pulmonary embolus?


    A.Fleischner sign


    B.Hampton hump


    C.Hoover sign


    D.Westermark sign


  Correct Answer C: Hoover sign. This sign indicates inward movement of the rib cage during inspiration and is associated with chronic obstructive pulmonary disease. Fleisher sign (indicative of an enlarged pulmonary artery), Hampton hump (peripheral wedge of airspace opacity due to pulmonary infarction), and Westermark sign (regional oligemia from vessel collapse distal to the pulmonary embolus) are all associated with pulmonary embolus.


193.Which patient would be the best candidate to receive succinylcholine as part of rapid sequence intubation?


    A.Patient brought in acutely with severe burns needing airway protection


    B.Patient with severe burns 36 hours ago


    C.Patient with a history of malignant hyperthermia


    D.Patient with penetrating ocular trauma


    E.Pediatric patient with unknown medical history


    F.Dialysis patient with last dialysis unknown


  Correct Answer A: Patient brought in acutely with severe burns needing airway protection. This patient is the best candidate for succinylcholine. Option B is not indicated due to the increased risk of rhabdomyolysis and hyperkalemia in this patient. Succinylcholine can precipitate malignant hyperthermia (C) and also increases intraocular pressure (D). Option F is not the best choice due to the risk of hyperkalemia. There has been shown to be an increased risk of rhabdomyolysis/hyperkalemia/arrhythmia in pediatric patients with undiagnosed skeletal muscle disease; so it is avoided in pediatric populations.


194.Which of the following is a contraindication to cricothyroidotomy?


    A.Angioedema


    B.Oral trauma requiring airway protection


    C.History of laryngeal cancer with resection and radiation


    D.Pediatric trauma patient age 7 requiring airway protection


    E.Patient with recent mandibular open reduction internal fixation and needing airway protection


  Correct Answer D: Pediatric trauma patient age 7 requiring airway protection. The pediatric patient is the only absolute contraindication to open cricothyroidotomy. There is controversy about the age, but generally in patients under 12, a needle cricothyrotomy is indicated given the small cricothyroid membrane in infant and pediatric patients along with their funnel-shaped larynx. The other patients will be difficult to intubate with rapid sequence intubation, and cricothyroidotomy would be the surgical airway of choice if necessary.


195.What is the most serious complication of the below injury:



images


FIGURE 20.1. Radiograph reveals a complete posterolateral dislocation of the elbow.


(From Browner BD. Skeletal Trauma. 4th ed. Philadelphia, PA: W. B. Saunders; 2008. MD Consult.)

Only gold members can continue reading. Log In or Register to continue

Stay updated, free articles. Join our Telegram channel

Oct 8, 2016 | Posted by in EMERGENCY MEDICINE | Comments Off on Questions and Answers

Full access? Get Clinical Tree

Get Clinical Tree app for offline access